Difference between revisions of "New problems"

From Universe in Problems
Jump to: navigation, search
Line 2: Line 2:
  
  
<!--
 
<p style= "color: #999;font-size: 14px">New from 13-01-2015</p>
 
 
 
 
<div id="1301_1"></div>
 
<div style="border: 1px solid #AAA; padding:5px;">
 
'''Problem 1'''
 
<p style= "color: #999;font-size: 11px">problem id: 1301_1</p>
 
Show that for a spatially flat Universe consisting of one component with equation of state $p=w\rho$ the deceleration parameter is equal to $q=(1+3w)/2$.
 
<div class="NavFrame collapsed">
 
  <div class="NavHead">solution</div>
 
  <div style="width:100%;" class="NavContent">
 
    <p style="text-align: left;">\[\frac{\ddot a}{a}=-\frac{4\pi G}{3}\rho(1+3w).\]
 
In the flat case \[\rho=\frac{3}{8\pi G}H^2\] and \[q=-\frac{\dot a}{aH^2}=\frac12(1+3w).\]</p>
 
  </div>
 
</div></div>
 
 
 
<div id="1301_2"></div>
 
<div style="border: 1px solid #AAA; padding:5px;">
 
'''Problem 2'''
 
<p style= "color: #999;font-size: 11px">problem id: 1301_2</p>
 
Find generalization of the relation $q=(1+3w)/2$ for the non-flat case.
 
<div class="NavFrame collapsed">
 
  <div class="NavHead">solution</div>
 
  <div style="width:100%;" class="NavContent">
 
    <p style="text-align: left;">\[\frac{\ddot a}{a}=-\frac{4\pi G}{3}\rho(1+3w).\]
 
Substitution \[\rho=\frac{3}{8\pi G}\left(H^2+\frac k{a^2}\right)\] gives \[q=-\frac{\dot a}{aH^2}=\frac12(1+3w)\left(1+\frac k{a^2H^2}\right).\]</p>
 
  </div>
 
</div></div>
 
 
 
<div id="1301_3"></div>
 
<div style="border: 1px solid #AAA; padding:5px;">
 
'''Problem 3'''
 
<p style= "color: #999;font-size: 11px">problem id: 1301_3</p>
 
Find deceleration parameter for multi-component non-flat Universe.
 
<div class="NavFrame collapsed">
 
  <div class="NavHead">solution</div>
 
  <div style="width:100%;" class="NavContent">
 
    <p style="text-align: left;">In this case
 
\[q=-\frac{\dot a}{aH^2}=\frac12(1+3\frac p\rho)\left(1+\frac k{a^2H^2}\right) =\] \[=\frac12\left[1+3\frac p\rho\left(1+\frac k{a^2H^2}\right)\right]+\frac k{2a^2H^2},\]
 
\[1+\frac k{a^2H^2}=\sum\limits_i\Omega_i,\quad\frac p\rho=\frac{\sum\limits_iw_i\Omega_i}{\sum\limits_i\Omega_i},\]
 
\[q=\frac12\left(1+3\sum\limits_i\Omega_i\right)+\frac k{2a^2H^2}.\]
 
</p>
 
  </div>
 
</div></div>
 
 
 
<div id="1301_4"></div>
 
<div style="border: 1px solid #AAA; padding:5px;">
 
'''Problem 4'''
 
<p style= "color: #999;font-size: 11px">problem id: 1301_4</p>
 
Show that the expression for deceleration parameter obtained in the previous problem can be presented in the following form \[q=\frac{\Omega_{total}}2+\frac32\sum\limits_i w_i \Omega_i.\]
 
<div class="NavFrame collapsed">
 
  <div class="NavHead">solution</div>
 
  <div style="width:100%;" class="NavContent">
 
    <p style="text-align: left;">Using results of the previous problem one obtains
 
\[q=\frac12\left(1+3\sum\limits_iw_i\Omega_i\right)+\frac k{2a^2H^2}=\frac12\left(1+\frac k{a^2H^2}\right)+\frac32\sum\limits_iw_i\Omega_i=\]
 
\[=\frac12\sum\limits_i\Omega_i+\frac32\sum\limits_iw_i\Omega_i=\frac{\Omega_{total}}2+\frac32\sum\limits_i w_i \Omega_i.\]</p>
 
  </div>
 
</div></div>
 
 
 
<div id="1301_5"></div>
 
<div style="border: 1px solid #AAA; padding:5px;">
 
'''Problem 5'''
 
<p style= "color: #999;font-size: 11px">problem id: </p>
 
Let us consider the model of two-component Universe [ J. Ponce de Leon, cosmological model with variable equations of state for matter and dark energy, arXiv:1204.0589]. Such approximation is sufficient to achieve good accuracy on each stage of its evolution. At the present time the two dark components - dark matter and dark energy - are considered to be dominating. We neglect meanwhile the interaction between the components and as a result they separately satisfy the conservation equation. Let us assume that the state equation parameter for each component depends on the scale factor
 
\begin{align}
 
\nonumber p_{de} & = W(a) \rho_{de};\\
 
\nonumber p_m & = w(a)\rho_{m}.
 
\end{align}
 
Express the relative densities $\Omega_m$ and $\Omega_{de}$ in terms of the deceleration parameter and the state equation parameters $w(a)$ and $W(a)$.
 
<div class="NavFrame collapsed">
 
  <div class="NavHead">solution</div>
 
  <div style="width:100%;" class="NavContent">
 
    <p style="text-align: left;">In the considered case
 
\[\Omega_m +\Omega_{de}=1+\frac{k}{a^2H^2},\]
 
\[q=\frac12+\frac32\left[W\Omega_{de}+w\Omega_m\right]+\frac{k}{2a^2H^2}.\]
 
These can, formally, be regarded as two equations for $\Omega_{de}$ and $\Omega_{m}$. Solving them we get
 
\begin{align}
 
\nonumber \Omega_{m} & = \frac{2q-1-3W}{3(w-W)}-\frac{k(1+3W)}{3a^2H^2(w-W)};\\
 
\nonumber \Omega_{m} & = \frac{2q-1-3w}{3(w-W)}-\frac{k(1+3w)}{3a^2H^2(w-W)}.
 
\end{align}</p>
 
  </div>
 
</div></div>
 
 
 
<div id="1301_6"></div>
 
<div style="border: 1px solid #AAA; padding:5px;">
 
'''Problem 6'''
 
<p style= "color: #999;font-size: 11px">problem id: 1301_6</p>
 
Find the upper and lower limits on the deceleration parameter using results of the previous problem.
 
<div class="NavFrame collapsed">
 
  <div class="NavHead">solution</div>
 
  <div style="width:100%;" class="NavContent">
 
    <p style="text-align: left;">We note that the denominator in these expressions is always positive because $W<0$ for dark energy. Thus, the fact that $\Omega_{m(de)}>0$ imposes an upper and lower limit on $q$,
 
\[\frac{1+3W}{2}\left(1+\frac{k}{a^2H^2}\right)\le q\le \frac{1+3w}{2}\left(1+\frac{k}{a^2H^2}\right).\]</p>
 
  </div>
 
</div></div>
 
 
 
<div id="1301_7"></div>
 
<div style="border: 1px solid #AAA; padding:5px;">
 
'''Problem 7'''
 
<p style= "color: #999;font-size: 11px">problem id: 1301_7</p>
 
Find the relation between the total pressure and the deceleration parameter for the flat one-component Universe
 
<div class="NavFrame collapsed">
 
  <div class="NavHead">solution</div>
 
  <div style="width:100%;" class="NavContent">
 
    <p style="text-align: left;">It follows from the conservation equation that
 
\[p=-\frac{\dot\rho}{3H}\frac{w}{1+w}.\]
 
Using
 
\[w = \frac{2q-1}3;\quad \dot\rho=\frac{3}{4\pi G}H\dot H,\quad \dot H=-H^2(1+q)\]
 
one finds
 
\[p=\frac{H^2}{8\pi G}(2q-1).\]</p>
 
  </div>
 
</div></div>
 
 
 
<div id="1301_8"></div>
 
<div style="border: 1px solid #AAA; padding:5px;">
 
'''Problem 8'''
 
<p style= "color: #999;font-size: 11px">problem id: 1301_8</p>
 
The expansion of pressure by the cosmic time is given by
 
\[p(t)=\left.\sum\limits_{k=0}^\infty\frac1{k!}\frac{d^kp}{dt^k}\right|_{t=t_0}(t-t_0)^k.\]
 
Using cosmography parameters , evaluate the derivatives up to the fourth order.
 
<div class="NavFrame collapsed">
 
  <div class="NavHead">solution</div>
 
  <div style="width:100%;" class="NavContent">
 
    <p style="text-align: left;"></p>
 
  </div>
 
</div></div>
 
 
 
<div id="1301_9"></div>
 
<div style="border: 1px solid #AAA; padding:5px;">
 
'''Problem 9'''
 
<p style= "color: #999;font-size: 11px">problem id: 1301_9</p>
 
Show that for one-component flat Universe filled with ideal fluid of density $\rho$.
 
<div class="NavFrame collapsed">
 
  <div class="NavHead">solution</div>
 
  <div style="width:100%;" class="NavContent">
 
    <p style="text-align: left;">\[q=-1-\frac12\frac{d\ln\rho}{d\ln a}.\]</p>
 
  </div>
 
</div></div>
 
 
 
<div id="1301_10"></div>
 
<div style="border: 1px solid #AAA; padding:5px;">
 
'''Problem 10'''
 
<p style= "color: #999;font-size: 11px">problem id: 1301_10</p>
 
For what values of the state parameter $w$ the rate of expansion of a one-component flat Universe increases with time?
 
<div class="NavFrame collapsed">
 
  <div class="NavHead">solution</div>
 
  <div style="width:100%;" class="NavContent">
 
    <p style="text-align: left;"></p>
 
  </div>
 
</div></div>
 
 
 
<div id="1301_11"></div>
 
<div style="border: 1px solid #AAA; padding:5px;">
 
'''Problem 11'''
 
<p style= "color: #999;font-size: 11px">problem id: 1301_11</p>
 
Express the age of the spatially flat Universe filled with a single component with equation of state $p=w\rho$ through the deceleration parameter.
 
<div class="NavFrame collapsed">
 
  <div class="NavHead">solution</div>
 
  <div style="width:100%;" class="NavContent">
 
    <p style="text-align: left;">For spatially flat one-component Universe with state equation $p=w\rho$ the scale factor is \[a\propto t^\frac{2}{3(1+w)}\] and therefore \[H=\frac{2}{3(1+w)}\frac1t.\]
 
 
Using $q=(1+3w)/2$ one can find a simple relation between the current age of the Universe and the DP
 
\[t_0=\frac{H_0^{-1}}{1+q}.\]</p>
 
  </div>
 
</div></div>
 
 
 
<div id="1301_12"></div>
 
<div style="border: 1px solid #AAA; padding:5px;">
 
'''Problem 12'''
 
<p style= "color: #999;font-size: 11px">problem id: 1301_12</p>
 
Suppose we know the current values of the Hubble constant $H_0$ and the deceleration parameter $q_0$ for a closed Universe filled with dust only. How many times larger will  it ever become? Find lifetime of such a Universe.
 
<div class="NavFrame collapsed">
 
  <div class="NavHead">solution</div>
 
  <div style="width:100%;" class="NavContent">
 
    <p style="text-align: left;"></p>
 
  </div>
 
</div></div>
 
 
 
<div id="1301_13"></div>
 
<div style="border: 1px solid #AAA; padding:5px;">
 
'''Problem 13'''
 
<p style= "color: #999;font-size: 11px">problem id: 1301_13</p>
 
In a closed Universe filled with non-relativistic matter the current values of the Hubble constant is $H_0$, the deceleration parameter is $q_0$. Find the current age of this Universe.
 
<div class="NavFrame collapsed">
 
  <div class="NavHead">solution</div>
 
  <div style="width:100%;" class="NavContent">
 
    <p style="text-align: left;"></p>
 
  </div>
 
</div></div>
 
 
 
<div id="1301_14"></div>
 
<div style="border: 1px solid #AAA; padding:5px;">
 
'''Problem 14'''
 
<p style= "color: #999;font-size: 11px">problem id: 1301_14</p>
 
In a closed Universe filled with dust the current value of the Hubble constant is $H_0$ and of the deceleration parameter $q_0$.
 
 
a) What is the total  proper volume of the Universe at present time?
 
 
b) What is the total  current proper volume of space occupied by matter which we are presently observing?
 
 
c) What is the total  proper volume of space which we are directly observing?
 
<div class="NavFrame collapsed">
 
  <div class="NavHead">solution</div>
 
  <div style="width:100%;" class="NavContent">
 
    <p style="text-align: left;"></p>
 
  </div>
 
</div></div>
 
 
 
<div id="1301_15"></div>
 
<div style="border: 1px solid #AAA; padding:5px;">
 
'''Problem 15'''
 
<p style= "color: #999;font-size: 11px">problem id: 1301_15</p>
 
For the closed ($k=+1$) model of Universe, filled with non-relativistic matter, show that solutions of the Friedmann equations can be represented in terms of the two parameters $H_0$ and $q_0$. [Y.Shtanov, Lecture Notes on theoretical cosmology, 2010 ]
 
<div class="NavFrame collapsed">
 
  <div class="NavHead">solution</div>
 
  <div style="width:100%;" class="NavContent">
 
    <p style="text-align: left;">The Friedman equations can then be represented in the form
 
\begin{align}
 
\label{background_2_29} H^2 & +\frac1{a^2} =\frac{8\pi G}3\rho,\\
 
\nonumber 2\frac{\ddot a}{a} & +H^2+\frac1{a^2} = 0.
 
\end{align}
 
Using this equations one can find the relations between the current values of the Universe's parameters
 
\begin{align}
 
\label{background_2_30} H_0^2 & =\frac1{a_0^2(2q_0-1)},\\
 
\nonumber q_0 & = \frac{4\pi G}{3H_0^2}\rho_0.
 
\end{align}
 
Note that in general
 
\[q_0=\frac{4\pi G}3\frac{\rho_0+3p_0}{H_0^2}=\frac{\rho_0+3p_0}{2\rho_{0,crit}},\quad \rho_{0,crit}=\frac{3H_0^2}{8\pi G}.\]
 
For Universe filled only with non-relativistic matter one has $\Omega_m = 2q_0$. It is easy to see that $q_0>1/2$ and $\Omega_m>1$ as was expected in the closed model. Using (\ref{background_2_30}) one can rewrite the equation for scale factor in the form
 
\begin{equation}\label{background_2_31}\dot a^2=\frac\alpha a-1,\quad \alpha\equiv\frac{2q_0}{H_0(2q_0-1)^{3/2}}.\end{equation}
 
It is easy to see that the considered model includes both parameters $H_0$ and $q_0$. integration of (\ref{background_2_31}) gives \[t=\int \sqrt{\frac{a}{\alpha-a}}\,da.\]
 
Substitution \[a=\frac\alpha2(1-\cos\tau)\] leads to \begin{equation}\label{background_2_33}t=\frac\alpha2(\tau-\sin\tau).\end{equation}
 
Because of the relation $dt=ad\tau$ it is evident that the variable $\tau$ is the conformal time. Taking the constants of integration so that $a=0$ as $t=0$ (and $\tau=0$) we can see that $a=a_0$ at $\tau=\tau_0$. Consequently, \[\cos\tau_0=\frac{1-q_0}{q_0},\quad \sin\tau_0=\frac{\sqrt{2q_0-1}}{q_0}.\]
 
From (\ref{background_2_33}) it follows that the age of Universe in close model is \[t_0=\frac\alpha2(\tau_0-\sin\tau_0)=\frac{q_0}{H_0(2q_0-1)^{3/2}}\left(\arccos{\frac{1-q_0}{q_0}} - \frac{\sqrt{2q_0-1}}{q_0}\right).\]
 
The maximum of scale factor reaches at $\tau=\pi$,
 
\[a_{\rm max}=\alpha=\frac{2q_0}{H_0(2q_0-1)^{3/2}}.\]
 
As one can see, all parameters of the model can be expressed in terms of parameters $H_0$ and $q_0$.</p>
 
  </div>
 
</div></div>
 
 
 
<div id="1301_16"></div>
 
<div style="border: 1px solid #AAA; padding:5px;">
 
'''Problem 16'''
 
<p style= "color: #999;font-size: 11px">problem id: 1301_16</p>
 
Do the same as in the previous problem for the case of open ($k=-1$) model of Universe.
 
<div class="NavFrame collapsed">
 
  <div class="NavHead">solution</div>
 
  <div style="width:100%;" class="NavContent">
 
    <p style="text-align: left;">In the case of open Universe the formulae (\ref{background_2_29})-(\ref{background_2_31}) transform into
 
\begin{align}
 
\label{background_2_37} H^2 & -\frac1{a^2} =\frac{8\pi G}3\rho,\\
 
\nonumber 2\frac{\ddot a}{a} & +H^2-\frac1{a^2} = 0,\\
 
\label{background_2_38} H_0^2 & =\frac1{a_0^2(1-2q_0)},\\
 
\nonumber q_0 & = \frac{4\pi G}{3H_0^2}\rho_0,
 
\end{align}
 
\begin{equation}\label{background_2_39}\dot a^2=\frac\beta a+1,\quad \beta\equiv\frac{2q_0}{H_0(1-2q_0)^{3/2}}.\end{equation}
 
Now $q_0<1/2$, $0\le\Omega_m< 1$. The solution of (\ref{background_2_39}) in the conformal time parametrization is \[a=\frac\beta2(\cosh\tau-1),\quad t=\frac\beta2(\sinh\tau-\tau).\] Current value $\tau_0$ of the conformal time is defined by the relation \[\cosh\tau_0=\frac{1-q_0}{q_0}.\]
 
The age of the Universe is
 
\[t_0=\frac\beta2(\sinh\tau_0-\tau_0) = \frac{q_0}{H_0(1-q_0)^{3/2}}\left(\frac{\sqrt{1-2q_0}}{q_0} - \ln\frac{1-q_0+\sqrt{1-2q_0}}{q_0}\right).\]
 
In this case again all characteristics of the model are expressed in terms of the parameters $H_0$ and $q_0$.</p>
 
  </div>
 
</div></div>
 
 
 
=Classification of models of Universe based on the deceleration parameter=
 
 
<div id="1301_17"></div>
 
<div style="border: 1px solid #AAA; padding:5px;">
 
'''Problem 17'''
 
<p style= "color: #999;font-size: 11px">problem id: 1301_17</p>
 
When the rate of expansion never changes, and $\dot a$ is constant, the scaling factor is proportional to time $t$, and the deceleration term is zero. When the Hubble parameter is constant, the deceleration parameter $q$ is also constant and equal to $-1$, as in the de Sitter model. All models can be characterized by whether they expand or contract, and accelerate or decelerate. Build a classification of such type using the signature of the Hubble parameter and the deceleration parameter.
 
<div class="NavFrame collapsed">
 
  <div class="NavHead">solution</div>
 
  <div style="width:100%;" class="NavContent">
 
    <p style="text-align: left;">a) $H>0$, $q>0$: expanding and decelerating<br/>
 
b) $H>0$, $q<0$: expanding and accelerating<br/>
 
c) $H<0$, $q>0$: contracting and decelerating<br/>
 
d) $H<0$, $q<0$: contracting and accelerating<br/>
 
e) $H>0$, $q=0$: expanding, zero deceleration<br/>
 
f) $H<0$, $q=0$: contracting, zero deceleration<br/>
 
g) $H=0$, $q=0$: static<br/>
 
 
 
There is little doubt that we live in an expanding Universe, and hence only (a), (b), and (e) are possible.  Evidences in favor of the fact that the expansion is presently accelerating continuously grows in number and therefore the current dynamics belongs to type (b).</p>
 
  </div>
 
</div></div>
 
 
 
<div id="1301_18"></div>
 
<div style="border: 1px solid #AAA; padding:5px;">
 
'''Problem 18'''
 
<p style= "color: #999;font-size: 11px">problem id: 1301_18</p>
 
Can the Universe variate its type of evolutions in frames of the classification given in the previous problem?
 
<div class="NavFrame collapsed">
 
  <div class="NavHead">solution</div>
 
  <div style="width:100%;" class="NavContent">
 
    <p style="text-align: left;">Of course, generally speaking, both the Hubble parameter and deceleration parameter can change their sign during the evolution. Therefore the evolving Universe can transit from one type to another. It is one of the basic tasks of cosmology to follow this evolution and clarify its causes.</p>
 
  </div>
 
</div></div>
 
 
 
<div id="1301_19"></div>
 
<div style="border: 1px solid #AAA; padding:5px;">
 
'''Problem 19'''
 
<p style= "color: #999;font-size: 11px">problem id: 1301_19</p>
 
Point out possible regime of expansion of Universe in the case of constant deceleration parameter.
 
<div class="NavFrame collapsed">
 
  <div class="NavHead">solution</div>
 
  <div style="width:100%;" class="NavContent">
 
    <p style="text-align: left;">In the case of constant deceleration parameter the Universe would exhibit decelerating expansion if $q>0$, an expansion with constant rate if $q=0$, accelerating power-law expansion if $-1<q<0$, exponential expansion (also known as de Sitter expansion) if $q=-1$ and super-exponential expansion if $q<-1$.</p>
 
  </div>
 
</div></div>
 
 
 
<div id="1301_20"></div>
 
<div style="border: 1px solid #AAA; padding:5px;">
 
'''Problem 20'''
 
<p style= "color: #999;font-size: 11px">problem id: 1301_20</p>
 
Having fixed the material content we can classify the model of Universe using the connection between the deceleration parameter and the spatial geometry. Perform this procedure in the case of Universe filled with non-relativistic matter.
 
<div class="NavFrame collapsed">
 
  <div class="NavHead">solution</div>
 
  <div style="width:100%;" class="NavContent">
 
    <p style="text-align: left;">
 
\begin{align}
 
\nonumber q>\frac12 & :\quad  closed\ spherical\ space & k&=+1;\\
 
\nonumber q=\frac12 & :\quad  open\ flat\ space & k&=0;\\
 
\nonumber q<\frac12 & :\quad  open\ hyperbolic\  space & k&=-1.
 
\end{align}</p>
 
  </div>
 
</div></div>
 
 
 
<div id="1301_21"></div>
 
<div style="border: 1px solid #AAA; padding:5px;">
 
'''Problem 21'''
 
<p style= "color: #999;font-size: 11px">problem id: 1301_21</p>
 
Models of the Universe can be classified basing on the relation between the deceleration parameter and age of the Universe. Build such a classification in the case of Universe filled with non-relativistic matter (the Einstein-de Sitter Universe)
 
<div class="NavFrame collapsed">
 
  <div class="NavHead">solution</div>
 
  <div style="width:100%;" class="NavContent">
 
    <p style="text-align: left;">For the Einstein-de Sitter Universe where the age is \[t^*=\frac23H^{-1}\] we  have
 
\begin{align}
 
\nonumber q>\frac12 & :\quad  age<t^*;\\
 
\nonumber q=\frac12 & :\quad  age=t^*;\\
 
\nonumber q<\frac12 & :\quad  age>t^*.
 
\end{align}</p>
 
  </div>
 
</div></div>
 
 
 
<div id="1301_22"></div>
 
<div style="border: 1px solid #AAA; padding:5px;">
 
'''Problem 22'''
 
<p style= "color: #999;font-size: 11px">problem id: 1301_22</p>
 
Show that sign of the deceleration parameter determines the difference between the actual age of the Universe and the Hubble time.
 
<div class="NavFrame collapsed">
 
  <div class="NavHead">solution</div>
 
  <div style="width:100%;" class="NavContent">
 
    <p style="text-align: left;">In a decelerating Universe with $q>0$, the age of the Universe will be less than the Hubble time, because at earlier times it was expanding at a faster rate, whereas a Universe that has always been accelerating, that is, $q<0$ for all time, will have an age that is greater than the Hubble time. A Universe that expands at a constant rate, $q=0$, has an age equal to the Hubble time.</p>
 
  </div>
 
</div></div>
 
 
 
<div id="1301_23"></div>
 
<div style="border: 1px solid #AAA; padding:5px;">
 
'''Problem 23'''
 
<p style= "color: #999;font-size: 11px">problem id: 1301_23</p>
 
Suppose the flat Universe is filled with non-relativistic matter with density $\rho_m$ and some substance with equation of state $p_X=w\rho_X$. Express the deceleration parameter through the ratio $r\equiv\rho_m/\rho_X$.
 
<div class="NavFrame collapsed">
 
  <div class="NavHead">solution</div>
 
  <div style="width:100%;" class="NavContent">
 
    <p style="text-align: left;">\[q=-\frac{\ddot a/a}{H^2}=\frac{\frac{4\pi G}{3}(\rho+3p)}{\frac{8\pi G}{3}\rho}=\frac12+\frac32\frac p\rho= \frac12+\frac32\frac{w\rho_X}{\rho_m+\rho_X} =\frac12+\frac32\frac w{(1+r)}.\]</p>
 
  </div>
 
</div></div>
 
 
 
<div id="1301_23_1"></div>
 
<div style="border: 1px solid #AAA; padding:5px;">
 
'''Problem 24'''
 
<p style= "color: #999;font-size: 11px">problem id: 1301_23_1</p>
 
Obtain Friedmann equations for the case of spatially flat $n$-dimensional Universe. (see Shouxin Chen, Gary W. Gibbons, Friedmann's Equations in All Dimensions and  Chebyshev's Theorem, arXiv: 1409.3352)
 
<div class="NavFrame collapsed">
 
  <div class="NavHead">solution</div>
 
  <div style="width:100%;" class="NavContent">
 
    <p style="text-align: left;">Consider an $(n+1)$-dimensional homogeneous and isotropic
 
Lorentzian spatially flat
 
spacetime with the metric
 
\begin{equation} \label{1}
 
ds^2=g_{\mu\nu} dx^\mu dx^\nu=-dt^2+a^2(t)g_{ij} dx^i dx^j,\quad i,j=1,\dots,n,
 
\end{equation}
 
where $t$ is the cosmological (or cosmic) time and $g_{ij}$ is the  metric of an $n$-dimensional Riemannian manifold
 
$M$ of constant scalar curvature characterized by an indicator, $k=-1,0,1$,
 
so that $M$ is an $n$-hyperboloid, the flat space $\Bbb R^n$, or an $n$-sphere, with the
 
respective metric
 
\begin{equation} \label{2}
 
g_{ij}dx^i dx^j=\frac1{1-kr^2}\,dr^2+r^2\,d\Omega^2_{n-1},
 
\end{equation}
 
where $r>0$ is the radial variable and $d\Omega_{n-1}^2$ denotes the canonical metric of the unit
 
sphere $S^{n-1}$ in $\Bbb R^n$. Inserting the metric (\ref{1})--(\ref{2}) into the Einstein equations
 
\begin{equation}
 
G_{\mu\nu}+\Lambda g_{\mu\nu}=8\pi G T_{\mu\nu},
 
\end{equation}
 
where $G_{\mu\nu}$ is the Einstein tensor, $G$ the universal gravitational constant, and $\Lambda$ the cosmological constant, the speed of light is set to unity, and $T_{\mu\nu}$ is the
 
energy-momentum tensor of an ideal cosmological fluid given by
 
\begin{equation} \label{4}
 
T^{\mu\nu}=\mbox{\footnotesize    diag}\{\rho_m,p_m,\dots,p_m\},
 
\end{equation}
 
with $\rho_m$ and $p_m$ the $t$-dependent matter energy density and pressure,
 
we arrive
 
at the Friedmann equations
 
\begin{align}
 
H^2=&\frac{16\pi G}{n(n-1)}\rho-\frac k{a^2},\label{5}\\
 
\dot{H}=&-\frac{8\pi G}{n-1}(\rho+p)+\frac k{a^2},\label{6}
 
\end{align}
 
in which $\rho,p$ are the effective energy
 
density and pressure related to $\rho_m,p_m$ through
 
\begin{equation} \label{8}
 
\rho=\rho_m+\frac{\Lambda}{8\pi G},\quad p=p_m-\frac{\Lambda}{8\pi G}.
 
\end{equation}
 
On the other hand, recall that, with (\ref{1}) and (\ref{4}) and (\ref{8}), the energy-conservation law, $\nabla_\nu T^{\mu\nu}=0$, takes the form
 
\begin{equation} \label{9}
 
\dot{\rho}_m+n(\rho_m+p_m)H=0.
 
\end{equation}</p>
 
  </div>
 
</div></div>
 
 
 
<div id="1301_23_2"></div>
 
<div style="border: 1px solid #AAA; padding:5px;">
 
'''Problem 25'''
 
<p style= "color: #999;font-size: 11px">problem id: 1301_23_2</p>
 
Analyze exact solutions of the Friedmann equations obtained in the previous problem in the case of flat ($k=0$) $n$-dimensional Universe filled with a barotropic liquid with the state equation
 
\begin{equation} \label{10}
 
p_m=w \rho_m.
 
\end{equation}
 
and obtain corresponding explicit expressions for the deceleration parameter.
 
<div class="NavFrame collapsed">
 
  <div class="NavHead">solution</div>
 
  <div style="width:100%;" class="NavContent">
 
    <p style="text-align: left;">Inserting (\ref{10}) into (\ref{9}), we have
 
\begin{equation}\label{11}
 
\dot{\rho}_m+n(1+w)\rho_m \frac{\dot{a}}a=0,
 
\end{equation}
 
which can be integrated to yield
 
\begin{equation}\label{12}
 
\rho_m=\rho_0 a^{-n(1+w)},
 
\end{equation}
 
where $\rho_0>0$ is an integration constant. Using (\ref{12}) in (\ref{8}), we arrive at
 
the relation
 
\begin{equation}\label{13}
 
\rho=\rho_0 a^{-n(1+w)}+\frac{\Lambda}{8\pi G}.
 
\end{equation}
 
From (\ref{5}) and (\ref{13}), we get the following equation of motion for the scale factor $a$:
 
\begin{equation} \label{14}
 
\dot{a}^2=\frac{16\pi G\rho_0}{n(n-1)}a^{-n(1+w)+2}+\frac{2\Lambda}{n(n-1)} a^2-k.
 
\end{equation}
 
 
To integrate (\ref{14}), we recall Chebyshev's  theorem: For rational numbers
 
$p,q,r$ ($r\neq0$) and nonzero real numbers $\alpha,\beta$, the integral \[\int x^p(\alpha+\beta x^r)^q\,dx\] is elementary
 
if and only if at least one of the quantities
 
\begin{equation}\label{cd}
 
\frac{p+1}r,\quad q,\quad \frac{p+1}r+q,
 
\end{equation}
 
is an integer. Another way to see the validity of the Chebyshev theorem is to
 
represent the integral of concern by a hypergeometric function such that when a quantity in (\ref{cd}) is an integer the hypergeometric function
 
is reduced into an elementary function. Consequently, when $k=0$ or $\Lambda=0$, and $w$ is rational, the Chebyshev theorem enables us to know that,
 
for exactly
 
what values of $n$ and $w$, the equation (\ref{14}) may be integrated.
 
 
For the spatially flat situation $k=0$ we rewrite equation (\ref{14}) as
 
\begin{equation} \label{15}
 
\dot{a}=\pm\sqrt{c_0 a^{-n(1+w)+2}+\Lambda_0 a^2},\quad c_0=\frac{16\pi G\rho_0}{n(n-1)},\quad\Lambda_0=\frac{2\Lambda}{n(n-1)}.
 
\end{equation}
 
Then (\ref{15}) reads
 
\begin{equation} \label{15a}
 
\pm\int a^{-1}\left(c_0 a^{-n(1+w)}+\Lambda_0 \right)^{-\frac12}da=t+C.
 
\end{equation}
 
It is clear that the integral on the left-hand side of (\ref{15a}) satisfies the integrability condition stated in the Chebyshev theorem for any $n$ and any rational $w$.
 
 
It turns out
 
that (\ref{15}) might be integrated for any real $w$ as well,
 
not necessarily rational.
 
To do so we apply $a>0$ and get from (\ref{15}) the equation
 
\begin{equation} \label{16}
 
\frac{d}{dt}\ln a=\pm\sqrt{c_0 a^{-n(1+w)}+\Lambda_0},
 
\end{equation}
 
or equivalently,
 
\begin{equation}\label{17}
 
\dot{u}=\pm\sqrt{c_0 e^{-n(1+w)u}+\Lambda_0},\quad u=\ln a.
 
\end{equation}
 
Set
 
\begin{equation}\label{18}
 
\sqrt{c_0 e^{-n(1+w)u}+\Lambda_0}=v.
 
\end{equation}
 
Then
 
\begin{equation}\label{19}
 
u=\frac{\ln c_0}{n(1+w)}-\frac 1{n(1+w)} \ln(v^2-\Lambda_0).
 
\end{equation}
 
Inserting (\ref{19}) into (\ref{17}), we find
 
\begin{equation}\label{20}
 
\dot{v}=\mp\frac12 n(1+w)(v^2-\Lambda_0),
 
\end{equation}
 
whose integration gives rise to the expressions
 
\begin{equation} \label{21}
 
v(t)=\left\{\begin{array}{rl} &v_0\left(1\pm \frac12 n(1+w)v_0t\right)^{-1},\quad \Lambda_0=0;\\
 
&\\
 
& \sqrt{\Lambda_0}(1+C_0 e^{\mp n(1+w)\sqrt{\Lambda_0} t})(1-C_0 e^{\mp n(1+w)\sqrt{\Lambda_0} t})^{-1},\\& C_0=(v_0-\sqrt{\Lambda_0})(v_0+\sqrt{\Lambda_0})^{-1},\quad \Lambda_0>0;\\
 
&\\
 
&\sqrt{-\Lambda_0}\tan\left(\mp\frac12 n(1+w)\sqrt{-\Lambda_0} t +\arctan\frac{v_0}{\sqrt{-\Lambda_0}}\right),\quad
 
\Lambda_0<0,
 
\end{array}
 
\right.
 
\end{equation}
 
where $v_0=v(0)$. Hence, in terms of $v$, we obtain the time-dependence of the scale factor $a$:
 
\begin{equation} \label{22}
 
a^{n(1+w)}(t)=\frac{8\pi G \rho_0}{\frac12 n(n-1)v^2(t)-\Lambda}.
 
\end{equation}
 
 
We now assume
 
\(w>-1\) in the equation of state in our subsequent discussion.
 
We are interested in solutions satisfying
 
\(a(0)=0.\)
 
 
When $\Lambda=0$, we combine (\ref{21}) and (\ref{22}) to get
 
\begin{equation} \label{x1}
 
a^{n(1+w)}(t)=4\pi G\rho_0\left(\frac n{n-1}\right)(1+w)^2 t^2.
 
\end{equation}
 
When $\Lambda>0$, we similarly obtain
 
\begin{equation}\label{x2}
 
a^{n(1+w)}(t)=\frac{8\pi G\rho_0}{\Lambda}\sinh^2\left(\sqrt{\frac{n\Lambda}{2(n-1)}}(1+w) t\right).
 
\end{equation}
 
When $\Lambda<0$ we rewrite (\ref{22}) as
 
\begin{equation} \label{23}
 
a^{n(1+w)}(t)=\frac{8\pi G \rho_0}{(-\Lambda)}\cos^2\left(\sqrt{\frac{n(-\Lambda)}{2(n-1)} }(1+w)t \mp\arctan\sqrt{\frac{n(n-1)}{-2\Lambda}}\, v_0\right).
 
\end{equation}
 
 
If we require $a(0)=0$, then (\ref{23}) leads to the conclusion
 
\begin{equation} \label{24}
 
a^{n(1+w)}(t)=\frac{8\pi G \rho_0}{(-\Lambda)}\sin^2\sqrt{\frac{n(-\Lambda)}{2(n-1)} }(1+w)t,
 
\end{equation}
 
which gives rise to a periodic Universe so that the scale factor $a$ reaches its maximum $a_m$,
 
\begin{equation} \label{25}
 
a^{n(1+w)}_m=\frac{8\pi G \rho_0}{(-\Lambda)},
 
\end{equation}
 
at the times
 
\begin{equation}\label{26}
 
t=t_{m,k}=\left(\frac\pi2+k\pi\right)\frac1{(1+w)}\sqrt{\frac{2(n-1)}{n(-\Lambda)}},\quad k\in\Bbb Z,
 
\end{equation}
 
and shrinks to zero at the times
 
\begin{equation}\label{27}
 
t=t_{0,k}=\frac{k\pi}{(1+w)}\sqrt{\frac{2(n-1)}{n(-\Lambda)}},\quad k\in\Bbb Z.
 
\end{equation}
 
 
Using the equations (\ref{x1},\ref{x2},\ref{24}), one easily obtains the expressions for the DP
 
\begin{equation}
 
q(t)=\left\{\begin{array}{cc} \frac n2 (1+w)-1, & \Lambda_0=0;\\
 
&\\
 
\frac{n(1+w)}{2\cosh^2\left(\sqrt{\frac{n\Lambda}{2(n-1)}}(1+w)t\right)}-1, & \Lambda_0>0;\\
 
&\\
 
\frac{n(1+w)}{2\cos^2\left(\sqrt{\frac{n\Lambda}{2(n-1)}}(1+w)t\right)}-1, &
 
\Lambda_0<0.
 
\end{array}
 
\right.
 
\end{equation}</p>
 
  </div>
 
</div></div>
 
 
=Deceleration as a cosmographic parameter=
 
 
<div id="1301_24"></div>
 
<div style="border: 1px solid #AAA; padding:5px;">
 
'''Problem 26'''
 
<p style= "color: #999;font-size: 11px">problem id: 1301_24</p>
 
Make transition from the derivatives w.r.t. cosmological time to that w.r.t. conformal time in definitions of the Hubble parameter and the deceleration parameter.
 
<div class="NavFrame collapsed">
 
  <div class="NavHead">solution</div>
 
  <div style="width:100%;" class="NavContent">
 
    <p style="text-align: left;">\[H=\frac{a'}{a^2},\quad q=-\left(\frac{aa''}{a'^2}-1\right),\]
 
where prime denotes derivative  w.r.t. the conformal time.</p>
 
  </div>
 
</div></div>
 
 
 
<div id="1301_25"></div>
 
<div style="border: 1px solid #AAA; padding:5px;">
 
'''Problem 27'''
 
<p style= "color: #999;font-size: 11px">problem id: 1301_25</p>
 
Make Taylor expansion of the scale factor in time using the cosmographic parameters.
 
<div class="NavFrame collapsed">
 
  <div class="NavHead">solution</div>
 
  <div style="width:100%;" class="NavContent">
 
    <p style="text-align: left;">\begin{align}
 
\nonumber a(t)=&a_0\left[1+H_0(t-t_0)-\frac12q_0H_0^2(t-t_0)^2+ \frac1{3!}j_0H_0^3(t-t_0)^3\right.\\
 
\label{background_4_3} &\left.+\frac1{4!}s_0H_0^4(t-t_0)^4+\frac1{5!}l_0H_0^5(t-t_0)^5 + O\left((t-t_0)^6\right)\right].
 
\end{align}</p>
 
  </div>
 
</div></div>
 
 
 
<div id="1301_26"></div>
 
<div style="border: 1px solid #AAA; padding:5px;">
 
'''Problem 28'''
 
<p style= "color: #999;font-size: 11px">problem id: 1301_26</p>
 
Make Taylor expansion of the redshift in time using the cosmographic parameters.
 
<div class="NavFrame collapsed">
 
  <div class="NavHead">solution</div>
 
  <div style="width:100%;" class="NavContent">
 
    <p style="text-align: left;">\begin{align}
 
\nonumber 1+z=&\left[1+H_0(t-t_0)-\frac12q_0H_0^2(t-t_0)^2+ \frac1{3!}j_0H_0^3(t-t_0)^3\right.\\
 
\nonumber &\left.+\frac1{4!}s_0H_0^4(t-t_0)^4+\frac1{5!}l_0H_0^5(t-t_0)^5 + O\left((t-t_0)^6\right)\right]^{-1}.\\
 
\nonumber z=&H_0(t-t_0)+\left(1+\frac{q_0}2\right)H_0^2(t-t_0)^2+\cdots.
 
\end{align}</p>
 
  </div>
 
</div></div>
 
 
 
<div id="1301_27"></div>
 
<div style="border: 1px solid #AAA; padding:5px;">
 
'''Problem 29'''
 
<p style= "color: #999;font-size: 11px">problem id: 1301_27</p>
 
Show that \[q(t)=\frac{d}{dt}\left(\frac1H\right)-1.\]
 
<div class="NavFrame collapsed">
 
  <div class="NavHead">solution</div>
 
  <div style="width:100%;" class="NavContent">
 
    <p style="text-align: left;">\[q(t)=\frac{d}{dt}\left(\frac1H\right)-1=-\frac{\dot H}{H^2}-1=-\frac{\frac{\ddot a}{a}-H^2}{H^2}-1=q.\]</p>
 
  </div>
 
</div></div>
 
 
 
<div id="1301_28"></div>
 
<div style="border: 1px solid #AAA; padding:5px;">
 
'''Problem 30'''
 
<p style= "color: #999;font-size: 11px">problem id: 1301_28</p>
 
Show that the deceleration parameter as a function of the red shift satisfies the following relations
 
\begin{align}
 
\nonumber q(z)=&\frac{1+z}{H}\frac{d H}{dz}-1;\\
 
\nonumber q(z)=&\frac12(1+z)\frac1{H^2}\frac{d H^2}{dz}-1;\\
 
\nonumber q(z)=&\frac12\frac{d\ln H^2}{d\ln(1+z)}-1;\\
 
\nonumber q(z)=&\frac{d\ln H}{dz}(1+z)-1.
 
\end{align}
 
<div class="NavFrame collapsed">
 
  <div class="NavHead">solution</div>
 
  <div style="width:100%;" class="NavContent">
 
    <p style="text-align: left;"></p>
 
  </div>
 
</div></div>
 
 
 
<div id="1301_29"></div>
 
<div style="border: 1px solid #AAA; padding:5px;">
 
'''Problem 31'''
 
<p style= "color: #999;font-size: 11px">problem id: 1301_29</p>
 
Show that the deceleration parameter as a function of the scale factor satisfies the following relations
 
\begin{align}
 
\nonumber q(a)=&-\left(1+\frac{a\frac{dH}{da}}{H}\right);\\
 
\nonumber q(a)=&\frac{d\ln(aH)}{d\ln a}.
 
\end{align}
 
<div class="NavFrame collapsed">
 
  <div class="NavHead">solution</div>
 
  <div style="width:100%;" class="NavContent">
 
    <p style="text-align: left;"></p>
 
  </div>
 
</div></div>
 
 
 
<div id="1301_30"></div>
 
<div style="border: 1px solid #AAA; padding:5px;">
 
'''Problem 32'''
 
<p style= "color: #999;font-size: 11px">problem id: 1301_30</p>
 
Show that the derivatives $dH/dz$, $d^2H/dz^2$, $d^3H/dz^3$ and $d^4H/dz^4$ can be expressed through the deceleration parameter $q$ and other cosmological parameters.
 
<div class="NavFrame collapsed">
 
  <div class="NavHead">solution</div>
 
  <div style="width:100%;" class="NavContent">
 
    <p style="text-align: left;">\begin{align}
 
\frac{dH}{dz}=&\frac{1+q}{1+z}H;\\
 
\nonumber \frac{d^2H}{dz^2}=&\frac{j-q^2}{(1+z)^2}H;\\
 
\nonumber \frac{d^3H}{dz^3}=&\frac{H}{(1+z)^3}\left(3q^2+3q^3-4qj-3j-s\right);\\
 
\nonumber \frac{d^4H}{dz^4}=&\frac{H}{(1+z)^4}\left(-12q^2-24q^3-15q^4+32qj+25q^2j+7qs+12j-4j^2+8s+l\right).
 
\end{align}</p>
 
  </div>
 
</div></div>
 
 
 
<div id="1301_31"></div>
 
<div style="border: 1px solid #AAA; padding:5px;">
 
'''Problem 33'''
 
<p style= "color: #999;font-size: 11px">problem id: 1301_31</p>
 
Use results of the previous problem to make Taylor expansion of the Hubble parameter in redshift.
 
<div class="NavFrame collapsed">
 
  <div class="NavHead">solution</div>
 
  <div style="width:100%;" class="NavContent">
 
    <p style="text-align: left;">\[H(z)=H_0+\left.\frac{dH}{dz}\right|_{z=0}z +\frac12\left.\frac{d^2H}{dz^2}\right|_{z=0}z^2 +\frac16\left.\frac{d^3H}{dz^3}\right|_{z=0}z^3+\dots=\]
 
\[=H_0(1+(1+q_0)z+\frac12(j_0-q_0^2)z^2+\frac16(3q_0^2+3q_0^3-4q_0j_0-3j_0-s)z^3).\]
 
There is a decomposition for the inverse Hubble parameter
 
\begin{align}
 
\nonumber \frac{d}{dz}\left(\frac1H\right)=&-\frac1{H^2}\frac{dH}{dz}=-\frac{1+q}{1+z}\frac1H;\\
 
\nonumber \frac{d^2}{dz^2}\left(\frac1H\right)=& 2\left(\frac{1+q}{1+z}\right)^2\frac1H =\left(\frac{2+4q+3q^2-j}{(1+z)^2}\right)\frac1H ;\\
 
\nonumber \frac1{H(z)}=& \frac1{H_0}\left[1-(1+q_0)z+\frac{2+4q_0+3q_0^2-j_0}{6}z^2+\dots\right].
 
\end{align}</p>
 
  </div>
 
</div></div>
 
 
 
<div id="1301_32"></div>
 
<div style="border: 1px solid #AAA; padding:5px;">
 
'''Problem 34'''
 
<p style= "color: #999;font-size: 11px">problem id: 1301_32</p>
 
Express derivatives of the Hubble parameter squared w.r.t. the redshift $d^iH^2/dz^i$, $i=1,2,3,4$ in terms of the cosmographic parameters.
 
<div class="NavFrame collapsed">
 
  <div class="NavHead">solution</div>
 
  <div style="width:100%;" class="NavContent">
 
    <p style="text-align: left;">\begin{align}
 
\nonumber \frac{dH^2}{dz}=&\frac{2H^2}{1+z}(1+q),\\
 
\nonumber \frac{d^2H^2}{dz^2}=&\frac{2H^2}{(1+z)^2}(1+2q+j),\\
 
\nonumber \frac{d^3H^2}{dz^3}=&\frac{2H^2}{(1+z)^3}(-qj-s),\\
 
\nonumber \frac{d^4H^2}{dz^4}=&\frac{2H^2}{(1+z)^4}(4qj+3qs+3q^2j-j^2+4s+l).
 
\end{align}</p>
 
  </div>
 
</div></div>
 
 
 
<div id="1301_33"></div>
 
<div style="border: 1px solid #AAA; padding:5px;">
 
'''Problem 35'''
 
<p style= "color: #999;font-size: 11px">problem id: 1301_33</p>
 
Express the current values of deceleration and jerk parameters in terms of $N=-\ln(1+z)$.
 
<div class="NavFrame collapsed">
 
  <div class="NavHead">solution</div>
 
  <div style="width:100%;" class="NavContent">
 
    <p style="text-align: left;">\begin{align}
 
\nonumber q_0=& \left.-\frac1{H^2}\left\{\frac12\frac{d\left(H^2\right)}{dN}+H^2\right\}\right|_{N=0},\\
 
\nonumber j_0=&\left.\left\{\frac1{2H^2}\frac{d^2\left(H^2\right)}{dN^2} +\frac3{2H^2}\frac{d\left(H^2\right)}{dN}+1\right\}\right|_{N=0}.
 
\end{align}</p>
 
  </div>
 
</div></div>
 
 
 
<div id="1301_34"></div>
 
<div style="border: 1px solid #AAA; padding:5px;">
 
'''Problem 36'''
 
<p style= "color: #999;font-size: 11px">problem id: 1301_34</p>
 
Express time derivatives of the Hubble parameter in terms of the cosmographic parameters.
 
<div class="NavFrame collapsed">
 
  <div class="NavHead">solution</div>
 
  <div style="width:100%;" class="NavContent">
 
    <p style="text-align: left;">\begin{align}
 
\dot H=& -H^2(1+q),\\
 
\nonumber \ddot H=& H^3(j+3q+2),\\
 
\nonumber \dddot H=& H^4[s-4j-3q(q+4)-6],\\
 
\nonumber \ddddot H=& H^5[l-5s+10(q+2)j+30(q+2)q+24].
 
\end{align}</p>
 
  </div>
 
</div></div>
 
 
 
<div id="1301_35"></div>
 
<div style="border: 1px solid #AAA; padding:5px;">
 
'''Problem 37'''
 
<p style= "color: #999;font-size: 11px">problem id: 1301_35</p>
 
Express the deceleration parameter as power series in redshift $z$ or $y$-redshift $z/(1+z)$.
 
<div class="NavFrame collapsed">
 
  <div class="NavHead">solution</div>
 
  <div style="width:100%;" class="NavContent">
 
    <p style="text-align: left;">\begin{align}
 
\nonumber q(z)=&q_0 + \left(-q_0-2q_0^2+j_0\right)z+\frac12\left(2q_0+8q_0^2+8q_0^3-7q_0j_0-4j_0-s_0\right)z^2+O(z^3),\\
 
\nonumber q(y)=&q_0 +\left(-q_0-2q_0^2+j_0\right)y+\frac12\left(4q_0+8q_0^3-7q_0j_0-2j_0-s_0\right)y^2+O(y^3).
 
\end{align}</p>
 
  </div>
 
</div></div>
 
 
 
<div id="1301_36"></div>
 
<div style="border: 1px solid #AAA; padding:5px;">
 
'''Problem 38'''
 
<p style= "color: #999;font-size: 11px">problem id: 1301_36</p>
 
Show that the Hubble parameter is connected to the deceleration parameter by the integral relation \[H=H_0\exp\left[\int\limits_0^z[q(z')+1]d\ln (1+z')\right].\]
 
<div class="NavFrame collapsed">
 
  <div class="NavHead">solution</div>
 
  <div style="width:100%;" class="NavContent">
 
    <p style="text-align: left;"></p>
 
  </div>
 
</div></div>
 
 
 
<div id="1301_37"></div>
 
<div style="border: 1px solid #AAA; padding:5px;">
 
'''Problem 39'''
 
<p style= "color: #999;font-size: 11px">problem id: 1301_37</p>
 
Show that derivatives of the lower order parameters can be expressed through the higher ones, for instance \[\frac{dq}{d\ln(1+z)}=j-q(2q+1).\]
 
<div class="NavFrame collapsed">
 
  <div class="NavHead">solution</div>
 
  <div style="width:100%;" class="NavContent">
 
    <p style="text-align: left;">Differentiate the relation \[q(z)=\frac{1+z}{H}\frac{d H}{dz}-1\] to find
 
\[\frac{dq}{dz}=\frac1H\frac{dH}{dz}-\frac{1+z}{H^2}\left(\frac{dH}{dz}\right)^2+\frac{1+z}{H}\frac{d^2H}{dz^2}.\]
 
Using \[\frac{dH}{dz}=\frac{1+q}{1+z}H,\quad \frac{d^2H}{dz^2}=\frac{j-q^2}{(1+z)^2}H,\] one obtains \[\frac{dq}{d\ln(1+z)}=j-q(2q+1).\]</p>
 
  </div>
 
</div></div>
 
 
 
=Cosmological scalars and the Friedmann equation=
 
 
''Dunajski and  Gibbons [M. Dunajski, Gary Gibbons,  Cosmic Jerk, Snap and Beyond, arXiv:0807.0207] proposed an original way to test the General Relativity and the cosmological models based on it. The procedure implies expressing the Friedmann equation in terms of directly measurable cosmological scalars constructed out of higher derivatives of the scale factor, i.e. cosmographic parameters $H,q,j,s,l$. In other words, the key idea is to treat the Friedmann equations as one algebraic constraint between the scalars. This links the measurement of the cosmological parameters to a test of General Relativity, or any of its modifications.''
 
 
 
<div id="1301_38"></div>
 
<div style="border: 1px solid #AAA; padding:5px;">
 
'''Problem 40'''
 
<p style= "color: #999;font-size: 11px">problem id: 1301_38</p>
 
Express the curvature parameter $k$ terms of the cosmographic parameters for the case of Universe filled with non-interacting cosmological constant and non-relativistic matter.
 
<div class="NavFrame collapsed">
 
  <div class="NavHead">solution</div>
 
  <div style="width:100%;" class="NavContent">
 
    <p style="text-align: left;">Using the relation $\rho_m=M/a^3$ ($M=const$) we rewrite the first Friedmann equation in the form
 
\begin{equation}\label{1301_38_e1} \dot a^2+k=\frac13\frac M a+\frac13\Lambda a,\quad 8\pi G=1.\end{equation}
 
Then we differentiate the latter equation two times to find
 
\begin{align}
 
\label{1301_38_e2} \ddot a=&-\frac16\frac M{a^2}=\frac13\Lambda a,\\
 
\nonumber \dddot a=&-\frac13\frac{M\dot a}{a^3}=\frac13\Lambda\dot a.
 
\end{align}
 
Using the definitions of the cosmographic parameters
 
\[H=\frac{\dot a}{a},\quad q=-a\frac{\ddot a}{\dot a^2},\quad j=a^2\frac{\dddot a}{\dot a^3},\] we represent (\ref{1301_38_e1}) in the form
 
\begin{align}
 
\nonumber q &= \frac12A-B;\\
 
\nonumber j &= A+B;\\
 
\nonumber A &\equiv\frac13 \frac{M}{a^3H^2};\\
 
\nonumber B &\equiv\frac13 \frac{\Lambda}{H^2}.
 
\end{align}
 
Then we find
 
\begin{align}
 
\nonumber A &=\frac23(j+q);\\
 
\nonumber B &=\frac23(\frac12j-q).
 
\end{align}
 
The Friedmann equation (\ref{1301_38_e1}) in terms of the above introduced variables $A$ and $B$ takes on the form \[\frac k{a^2}=(A+B-1)H^2\] or \[k=a^2H^2(j-1).\]</p>
 
  </div>
 
</div></div>
 
 
 
<div id="1301_39"></div>
 
<div style="border: 1px solid #AAA; padding:5px;">
 
'''Problem 41'''
 
<p style= "color: #999;font-size: 11px">problem id: 1301_39</p>
 
Do the same as in the previous problem for the case of Universe filled with non-interacting non-relativistic matter $\rho_m=M_m/a^3$ and radiation $\rho_r=M_r/a^4$.
 
<div class="NavFrame collapsed">
 
  <div class="NavHead">solution</div>
 
  <div style="width:100%;" class="NavContent">
 
    <p style="text-align: left;">We represent the first Friedmann equation in the form
 
\begin{equation}\label{1301_39_e1} \frac{\dot a^2}{a^2}+\frac k{a^2}=\frac{M_m}{a^3}+\frac{M_r}{a^4},\quad \frac{8\pi G}{3}=1.\end{equation}
 
We twice differentiate the latter equation to find
 
\begin{align}
 
\label{1301_39_e2} \ddot a=&-\frac12\frac{M_m}{a^2}-\frac{M_r}{a^3},\\
 
\nonumber \dddot a=&\frac{M_m}{a^3}\dot a+3\frac{M_r}{a^4}\dot a.
 
\end{align}
 
Using definitions of the cosmographic parameters $q$ and $j$, one obtains
 
\begin{align}
 
\nonumber q &= \frac12A+B;\\
 
\nonumber j &= A+3B;\\
 
\nonumber A &\equiv\frac{M_m}{a^3H^2};\\
 
\nonumber B &\equiv\frac{M_r}{a^4H^2}.
 
\end{align}
 
Then we find
 
\begin{align}
 
\nonumber A &=-2j+6q;\\
 
\nonumber B &=j-2q.
 
\end{align}
 
The Friedmann equation (\ref{1301_39_e1}) in terms of the above introduced variables $A$ and $B$ takes on the form \[\frac k{a^2}=(A+B-1)H^2\] or in terms of the cosmographic parameters \[k=a^2H^2(4q-j-1).\]</p>
 
  </div>
 
</div></div>
 
 
 
<div id="1301_40"></div>
 
<div style="border: 1px solid #AAA; padding:5px;">
 
'''Problem '''
 
<p style= "color: #999;font-size: 11px">problem id: 1301_40</p>
 
Check the expressions for the curvature $k$ obtained in the previous problem for two cases: a) a flat Universe solely filled with non-relativistic matter; b) a flat Universe solely filled with radiation.
 
<div class="NavFrame collapsed">
 
  <div class="NavHead">solution</div>
 
  <div style="width:100%;" class="NavContent">
 
    <p style="text-align: left;">In the first case $B=0$, $q=1/2$. Then $A=1$ and $k=0$. Note that in this case $j=1$.
 
 
In the second case $A=0$, $q=1$. Then $B=1$, $k=0$. In that case $j=3$.</p>
 
  </div>
 
</div></div>
 
 
 
<div id="1301_41"></div>
 
<div style="border: 1px solid #AAA; padding:5px;">
 
'''Problem '''
 
<p style= "color: #999;font-size: 11px">problem id: 1301_41</p>
 
Find relation between the cosmographic parameters free of any cosmological parameter for the case of Universe considered in the problem [[#1301_38]].
 
<div class="NavFrame collapsed">
 
  <div class="NavHead">solution</div>
 
  <div style="width:100%;" class="NavContent">
 
    <p style="text-align: left;">Using expression for $\dddot a$ obtained in the problem \ref{1301_38}, one finds
 
\[\ddddot a=\frac M3\frac{\ddot a}{a^3}-M\frac{\dot a^2}{a^4}+\frac\Lambda3\ddot a.\]
 
For the snap parameter \[s\equiv a^3\frac{\ddddot a}{\dot a^4}\] one obtains \[s=-3(A+B)q-3A.\]
 
Substitution of the parameters
 
\begin{align}
 
\nonumber A &=\frac23(j+q);\\
 
\nonumber B &=\frac23(\frac12j-q).
 
\end{align}
 
introduced in the problem \ref{1301_38}, one finally finds
 
\[s+2(q+j)+qj=0.\]
 
This fourth order ODE is equivalent to the Friedmann equation and has an advantage that it  appears as a constraint on directly measurable quantities.</p>
 
  </div>
 
</div></div>
 
 
 
<div id="1301_42"></div>
 
<div style="border: 1px solid #AAA; padding:5px;">
 
'''Problem '''
 
<p style= "color: #999;font-size: 11px">problem id: 1301_42</p>
 
Perform the same procedure for the Chaplygin gas with the equation of state $p=-A/\rho$.
 
<div class="NavFrame collapsed">
 
  <div class="NavHead">solution</div>
 
  <div style="width:100%;" class="NavContent">
 
    <p style="text-align: left;">For small values of $a(t)$ density and pressure of the Chaplygin gas reduces to that of dust $\rho\propto a^{-3}$, and for large $a$ one gets the de Sitter Universe: $\rho=const$, $p=-\rho$. In between these two regimes one can use the approximation
 
\begin{equation}\label{scalar_5}\rho=\sqrt A+\frac{B}{\sqrt{2A}}a^{-6}.\end{equation}
 
Thus $\sqrt A$ plays the role of a cosmological constant. We insert this to the Friedmann equation with $\Lambda$ and follow the procedure of eliminating the constants by differentiation. This leads to an approximate constraint
 
\begin{equation}\label{scalar_6}s+5(q+j)+qj=0.\end{equation}</p>
 
  </div>
 
</div></div>
 
 
 
<div id="1301_43"></div>
 
<div style="border: 1px solid #AAA; padding:5px;">
 
'''Problem '''
 
<p style= "color: #999;font-size: 11px">problem id: 1301_43</p>
 
Perform the same procedure for the generalized Chaplygin gas with the equation of state $\rho=-A/\rho^\alpha$.
 
<div class="NavFrame collapsed">
 
  <div class="NavHead">solution</div>
 
  <div style="width:100%;" class="NavContent">
 
    <p style="text-align: left;">\begin{equation}\label{scalar_7}s+(3\alpha+2)(q+j)+qj=0.\end{equation}
 
Note that for $\alpha=1$ we reproduce the above obtained result for the Chaplygin gas, while $\alpha=0$ we return to the results obtained in the problem (\ref{1301_41}).
 
 
If we want to exclude the parameter $\alpha$ from the latter equation we must take one more derivative of the Friedmann equation and introduce an additional cosmological parameter \[l=\frac1a\frac{d^5a}{dt^5}\left(\frac1a\frac{da}{dt}\right)^{-5}.\] As a result one obtains
 
\begin{equation}\label{scalar_8}
 
-2qs-2jq^2-lq-2sj-3sq^2-j^2q-lj+s^2-3q^2j-qsj+j^3-2j^2q^2=0.
 
\end{equation}
 
This constraint is again approximate and is valid only in the regime where the higher order terms in the expansion of $\rho$ can be dropped.</p>
 
  </div>
 
</div></div>
 
 
= Averaging deceleration parameter =
 
 
 
''Since the deceleration parameter $q$ is a slowly varying quantity (e.g. $q = 1/2$ for matter-dominated case and $q = -1$ in the Universe dominated by dark energy in form of cosmological constant), then the useful information is contained in its time average value, which is very interesting to obtain without integration of the equations of motions for the scale factor. Let us see how it is possible [J.Lima, Age of the Universe, Average Deceleration Parameter and Possible Implications for the End of Cosmology, arXiv:0708.3414]. For that purpose let us define average value $\bar q$ of this parameter on time interval $[0,t_0]$ with the expression \[\bar q(t_0)=\frac1{t_0}\int\limits_0^{t_0}q(t)dt.\]''
 
 
 
<div id="1301_44"></div>
 
<div style="border: 1px solid #AAA; padding:5px;">
 
'''Problem '''
 
<p style= "color: #999;font-size: 11px">problem id: 1301_44</p>
 
Express current value of the average deceleration parameter in terms of the Hubble parameter.
 
<div class="NavFrame collapsed">
 
  <div class="NavHead">solution</div>
 
  <div style="width:100%;" class="NavContent">
 
    <p style="text-align: left;">Making use of the definition of the deceleration parameter
 
\[q(t)=-\frac{\ddot a a}{\dot a^2}=\frac{d}{dt}\left(\frac1H\right)-1,\]
 
it is easy to obtain
 
\[\bar q(t_0)=-1+\frac1{t_0H_0}.\]</p>
 
  </div>
 
</div></div>
 
 
 
<div id="1301_45"></div>
 
<div style="border: 1px solid #AAA; padding:5px;">
 
'''Problem '''
 
<p style= "color: #999;font-size: 11px">problem id: 1301_45</p>
 
Show that current age of the Universe depends solely on average value of the deceleration parameter.
 
<div class="NavFrame collapsed">
 
  <div class="NavHead">solution</div>
 
  <div style="width:100%;" class="NavContent">
 
    <p style="text-align: left;">Using results of the previous problem one finds
 
\begin{equation}\label{background_5_3}t_0=\frac{H_0^{-1}}{1+\bar q}.\end{equation}
 
Naturally current age of the Universe is proportional to $H_0^{-1}$, but the proportionality coefficient is solely determined by the average value of the deceleration parameter. It is worth noting that this purely kinematic result depends on curvature of the Universe, nor on number of components filling it, nor on the type of gravity theory used.</p>
 
  </div>
 
</div></div>
 
 
 
<div id="1301_46"></div>
 
<div style="border: 1px solid #AAA; padding:5px;">
 
'''Problem '''
 
<p style= "color: #999;font-size: 11px">problem id: 1301_46</p>
 
Show that the Hubble time $H_0)^{-1}$ represents a characteristic time scale for age of the Universe at any stage of the evolution.
 
<div class="NavFrame collapsed">
 
  <div class="NavHead">solution</div>
 
  <div style="width:100%;" class="NavContent">
 
    <p style="text-align: left;">As the average value of the deceleration parameter $\bar q$ is of order of unity then from (see result of the previous problem) \[t_0=\frac{H_0^{-1}}{1+\bar q}\]
 
it immediately follows that the Hubble time $H_0)^{-1}$ represents a characteristic time scale at any stage of the evolution of the Universe.</p>
 
  </div>
 
</div></div>
 
 
 
=Energy conditions in terms of the deceleration parameter=
 
 
 
Dynamic model-independent constraints on the kinematics of the Universe can further be obtained from the so-called energy conditions. These conditions, based on quite general physical principles, impose restrictions on the components of the energy-momentum tensor $T_{\mu\nu}$. In choosing a model for the medium (a model, but not the equation of state!), these conditions can be transformed into inequalities restricting the possible values of pressure and density of the medium. In terms of density and pressure the energy conditions take on the form
 
\begin{align}
 
\nonumber NEC&\Rightarrow & \rho+p\ge&0, & &\\
 
\nonumber WEC&\Rightarrow & \rho\ge&0,&\rho+p\ge&0,\\
 
\nonumber SEC&\Rightarrow & \rho+3p\ge&0,&\rho+p\ge&0,\\
 
\nonumber DEC&\Rightarrow & \rho\ge&0,&-\rho\le p\le&\rho.
 
\end{align}
 
Here, NEC, WEC, SEC, and DEC correspond to the zero, weak, strong, and dominant energy conditions. Because these conditions do not require any definite equation of state for the substance filling the Universe, they impose very simple and model-independent constraints on the behavior of the energy density and pressure. Hence, the energy conditions provide one of the possibilities for explaining the evolution of the Universe on the basis of quite general principles.
 
 
 
 
<div id="1301_47"></div>
 
<div style="border: 1px solid #AAA; padding:5px;">
 
'''Problem '''
 
<p style= "color: #999;font-size: 11px">problem id: 1301_47</p>
 
Express the energy conditions in terms of the scale factor and its derivatives.
 
<div class="NavFrame collapsed">
 
  <div class="NavHead">solution</div>
 
  <div style="width:100%;" class="NavContent">
 
    <p style="text-align: left;">\begin{align}
 
\nonumber NEC&\Rightarrow & -\frac{\ddot a}{a}+\frac{\dot a^2}{a^2}+\frac{k}{a^2}\ge&0,\\
 
\nonumber WEC&\Rightarrow & \frac{\dot a^2}{a^2}+\frac{k}{a^2}\ge&0,\\
 
\nonumber SEC&\Rightarrow & \frac{\ddot a}{a}\le&0,\\
 
\nonumber DEC&\Rightarrow & \frac{\ddot a}{a}+2\left[\frac{\dot a^2}{a^2}+\frac{k}{a^2}\right]\ge&0.
 
\end{align}</p>
 
  </div>
 
</div></div>
 
 
 
<div id="1301_48"></div>
 
<div style="border: 1px solid #AAA; padding:5px;">
 
'''Problem '''
 
<p style= "color: #999;font-size: 11px">problem id: 1301_48</p>
 
Transform the energy conditions to constraints on the deceleration parameter for the flat Universe.
 
<div class="NavFrame collapsed">
 
  <div class="NavHead">solution</div>
 
  <div style="width:100%;" class="NavContent">
 
    <p style="text-align: left;">\begin{align}
 
\label{background_6_5} NEC&\Rightarrow  q\ge1,\\
 
\nonumber SEC&\Rightarrow  q\ge0,\\
 
\nonumber DEC&\Rightarrow  q\le2.
 
\end{align}
 
There is no WEC among above conditions because it is always satisfied for arbitrary real $a(t)$.</p>
 
  </div>
 
</div></div>
 
 
 
<div id="1301_49"></div>
 
<div style="border: 1px solid #AAA; padding:5px;">
 
'''Problem '''
 
<p style= "color: #999;font-size: 11px">problem id: 1301_49</p>
 
Analyze the constraints on the regimes of accelerated and decelerated expansion of the Universe following from the energy conditions.
 
<div class="NavFrame collapsed">
 
  <div class="NavHead">solution</div>
 
  <div style="width:100%;" class="NavContent">
 
    <p style="text-align: left;">The conditions (\ref{background_6_5}) considered separately in principle allow a possibility for both decelerated ($q>0$) and accelerated ($q<0$) expansion of the Universe. The constraints by NEC in (\ref{background_6_5}) have clear sense: as follows from the second Friedmann equation, the inequality $\rho+3p\le0$ gives necessary condition for accelerated expansion of the Universe, i.e. the accelerated expansion of the Universe is possible only in presence of components with high negative pressure $p<-\rho/3$. The SEC excludes existence of such components. As a result, $q\ge0$ in this case. At the same time, NEC and DEC are compatible with the condition $p<-\rho/3$ and therefore they allow the regimes with $q<0$.
 
 
It worth noting that even before the discovery of the accelerated expansion of the Universe in 1997, Visser [M. Visser, Science 276, 88 (1997), M. Visser,  Phys. Rev. D 56, 7578(1997)] already concluded, basing on analysis of the energy conditions, that current observations suggest that SEC was violated sometime between the epoch of galaxy formation and the present. This implies that no possible combination of "normal" matter is capable of fitting the observational data.</p>
 
  </div>
 
</div></div>
 
 
 
=Distance-Deceleration Parameter Relations=
 
 
In cosmology there are many different and equally natural definitions of the notion of ''distance'' between two objects or events. In particular, the luminosity distance $d_L$ of an object at redshift $z$ is $d_L=(L/2\pi F)^{1/2}$, where $L$ is the bolometric luminosity for a given object and $F$ is the bolometric energy flux received from that object. The expression for the luminosity distance in a FLRW Universe is
 
\begin{equation}\label{distance_7_1}
 
d_L(z)=(1+z)\left\{
 
\begin{array}{lr}
 
R\sinh\left[\frac1{H_0R}\int\limits_0^z\frac{dz'}{E(z')}\right], & open\\
 
H_0^{-1}\int\limits_0^z\frac{dz'}{E(z')}, & flat\\
 
R\sin\left[\frac1{H_0R}\int\limits_0^z\frac{dz'}{E(z')}\right], & closed
 
\end{array}
 
\right.\end{equation}
 
(Here and below we set $c=1$). Here $R$ is the (comoving) radius of curvature of the open or closed Universe, $E=H/H_0$.
 
 
 
 
 
<div id="1301_50"></div>
 
<div style="border: 1px solid #AAA; padding:5px;">
 
'''Problem '''
 
<p style= "color: #999;font-size: 11px">problem id: 1301_50</p>
 
Represent the luminosity distance in the flat Universe in terms of the deceleration parameter.
 
<div class="NavFrame collapsed">
 
  <div class="NavHead">solution</div>
 
  <div style="width:100%;" class="NavContent">
 
    <p style="text-align: left;">\[d_L(z)=(1+z)\int\limits_0^z\frac{dz'}{H(z')}=(1+z)H_0\int\limits_0^z du\exp\left\{-\int\limits_0^u [1+q(v)]d[\ln(1+v)]\right\}.\]</p>
 
  </div>
 
</div></div>
 
 
 
<div id="1301_51"></div>
 
<div style="border: 1px solid #AAA; padding:5px;">
 
'''Problem '''
 
<p style= "color: #999;font-size: 11px">problem id: 1301_51</p>
 
Find expression for the luminosity distance up to terms of order of $z^2$.
 
<div class="NavFrame collapsed">
 
  <div class="NavHead">solution</div>
 
  <div style="width:100%;" class="NavContent">
 
    <p style="text-align: left;">\begin{equation}\label{distance_7_3}d_L=\frac{z}{H_0}\left[1+\left(\frac{1-q_0}{2}\right)z+O(z^2)\right],\end{equation}
 
where in the spatially flat case
 
\[q_0=\frac12\sum\limits_i\Omega_{i0}(1+3w_i).\]
 
</p>
 
  </div>
 
</div></div>
 
 
 
<div id="1301_52"></div>
 
<div style="border: 1px solid #AAA; padding:5px;">
 
'''Problem '''
 
<p style= "color: #999;font-size: 11px">problem id: 1301_52</p>
 
Find expression for the luminosity distance in the next order $O(z^3)$ in the redshift.
 
<div class="NavFrame collapsed">
 
  <div class="NavHead">solution</div>
 
  <div style="width:100%;" class="NavContent">
 
    <p style="text-align: left;">\begin{align}
 
\nonumber d_L(z)= & \frac{cz}{H_0}\left[
 
1+\frac12(1-q_0)z-\frac16(1-q_0-3q_0^2+j_0)z^2\right.\\
 
+&\left.\frac1{24}(2-2q_0-15q_0^2-15q_0^3+5j_0+10j_0q_0+s_0)z^3+O(z^4)\right].
 
\end{align}
 
As was expected, the latter decomposition contains the cosmographic parameters defined in terms of the higher order time derivatives of the scale factor ($j\propto d^3a/dt^3$).
 
</p>
 
  </div>
 
</div></div>
 
 
 
<div id="1301_53"></div>
 
<div style="border: 1px solid #AAA; padding:5px;">
 
'''Problem '''
 
<p style= "color: #999;font-size: 11px">problem id: 1301_53</p>
 
Calculate the luminosity distance in the Universe filled with non-relativistic matter (Einstein-de Sitter model ($k=0$)).
 
<div class="NavFrame collapsed">
 
  <div class="NavHead">solution</div>
 
  <div style="width:100%;" class="NavContent">
 
    <p style="text-align: left;">\begin{equation}\label{distance_7_5}d_L=a_0r_1(1+z),\quad r_1=\int\limits_{t_1}^{t_0}\frac{dt}{a(t)}.\end{equation}
 
In the considered case
 
\[r_1=\frac{3t_0}{a_0}\left[1-\left(\frac{t_1}{t_0}\right)^{1/3}\right],\quad 1+z=\frac{a_0}{a_1}=\left(\frac{t_0}{t_1}\right)^{2/3},\]
 
thus
 
\[r_1=\frac{3t_0}{a_0}\left[1-\left(1+z\right)^{-1/2}\right]=\frac{2}{a_0H_0} \left[1-\left(1+z\right)^{-1/2}\right].\]
 
We took into account here that in the Einstein-de Sitter model $h_0t_0=2/3$. For the luminosity distance one ultimately obtains
 
\[d_L=a_0r_1(1+z)=\frac2{H_0}\left(1+z-\sqrt{1+z}\right).\]</p>
 
  </div>
 
</div></div>
 
 
 
<div id="1301_54"></div>
 
<div style="border: 1px solid #AAA; padding:5px;">
 
'''Problem '''
 
<p style= "color: #999;font-size: 11px">problem id: 1301_54</p>
 
The expression (\ref{distance_7_1}) allows to find the luminosity distance given the function $H(z)$. Solve the inverse problem for the flat case: find the Hubble parameter as a function of the luminosity distance [S. Nesseris and J. Garcia-Bellido, Comparative analysis of model-independent methods for exploring the nature of dark energy, arXiv:1306.4885].
 
<div class="NavFrame collapsed">
 
  <div class="NavHead">solution</div>
 
  <div style="width:100%;" class="NavContent">
 
    <p style="text-align: left;">In the flat case, differentiating the expression
 
\[d_L(z)=(1+z)H_0^{-1}\int\limits_0^z\frac{dz'}{E(z')},\]
 
one obtains
 
\[\frac{d(d_l)}{dz}=\frac{1+z}{H(z)}+\frac{d_L(z)}{1+z}\Rightarrow H(z)=\frac{(1+z)^2}{d'_L(1+z)-d_L},\]
 
where prime denotes derivative with respect to redshift $z$.</p>
 
  </div>
 
</div></div>
 
 
 
<div id="1301_55"></div>
 
<div style="border: 1px solid #AAA; padding:5px;">
 
'''Problem '''
 
<p style= "color: #999;font-size: 11px">problem id: 1301_55</p>
 
Use result of the previous problem to express deceleration parameter a s a function of luminosity distance and its derivatives.
 
<div class="NavFrame collapsed">
 
  <div class="NavHead">solution</div>
 
  <div style="width:100%;" class="NavContent">
 
    <p style="text-align: left;">Using result of the previous problem, one finds
 
\[q(z)=-1\frac{\dot H}{H^2}=-1+(1+z)\frac{H'}{H}=1-\frac{(1+z)^2d''_L(z)}{d'_L(1+z)-d_L},\]
 
where prime denotes derivative with respect to redshift $z$.
 
 
The latter formula can be represented in the form
 
\[q(N)=-1-\frac{H'(N)}{H(N)}=1+\frac{d''_L(N)+d'_L(N)}{d'_L(N)+d_L(N)}.\]
 
Here primes denote derivatives with respect to $N\equiv\ln a=-\ln(1+z)$.
 
For arbitrary geometry
 
\[q(z)=\frac{1+\omega_K d_L(z)d'_L(z)/(1+z)}{1+\omega_K d_L^2(z)/(1+z)^2}-\frac{(1+z)^2d''_L(z)}{d'_L(z)(1+z)-d_L}.\]</p>
 
  </div>
 
</div></div>
 
 
 
=Horizons=
 
 
 
 
<div id="1301_56"></div>
 
<div style="border: 1px solid #AAA; padding:5px;">
 
'''Problem '''
 
<p style= "color: #999;font-size: 11px">problem id: 1301_56</p>
 
The Hubble sphere in general does not coincide with the horizons, except when it becomes degenerate with the particle horizon at $q=1$ and with event horizon at $q=-1$. Show that.
 
<div class="NavFrame collapsed">
 
  <div class="NavHead">solution</div>
 
  <div style="width:100%;" class="NavContent">
 
    <p style="text-align: left;">The component with $q=1$ represents the ultra-relativistic matter (radiation) for which $a\propto t^{1/2}$ and therefore $H=1/(2t)$. In this case the particle horizon is finite and equals to
 
\[L_p=a(t)\int\limits_0^t\frac{dt'}{a(t')}=2t=\frac1{H(t)}=R_H.\]
 
 
The component with $q=-1$ represents the cosmological constant for which $a\propto e^{Ht}$, $H=const$. In this case the particle horizon is absent because the origin of the Universe is ''moved'' to $t=-\infty$. The event horizon has the following size
 
\[L_e=a(t)\int\limits_t^\infty\frac{dt'}{a(t')}=\frac1H=R_H.\]</p>
 
  </div>
 
</div></div>
 
 
 
<div id="1301_57"></div>
 
<div style="border: 1px solid #AAA; padding:5px;">
 
'''Problem '''
 
<p style= "color: #999;font-size: 11px">problem id: 1301_57</p>
 
Show that the Hubble sphere contracts when $q<-1$, remains stationary when $q=-1$ and expands when $q>-1$.
 
<div class="NavFrame collapsed">
 
  <div class="NavHead">solution</div>
 
  <div style="width:100%;" class="NavContent">
 
    <p style="text-align: left;">\[\frac{d}{dt}(R_H)=c\frac{d}{dt}\left(\frac1H\right)= -\frac{c}{H^2}\left(\frac{\ddot a}{a}-\frac{\dot a^2}{a^2}\right)=c(1+q).\]
 
 
As one can see, the Hubble sphere contracts when $q<-1$, remains stationary when $q=-1$ and expands when $q>-1$.</p>
 
  </div>
 
</div></div>
 
 
 
<div id="1301_58"></div>
 
<div style="border: 1px solid #AAA; padding:5px;">
 
'''Problem '''
 
<p style= "color: #999;font-size: 11px">problem id: 1301_58</p>
 
Analyze correspondence between kinematics of the Hubble sphere and boundaries of the observable Universe for different expansion regimes.
 
<div class="NavFrame collapsed">
 
  <div class="NavHead">solution</div>
 
  <div style="width:100%;" class="NavContent">
 
    <p style="text-align: left;">In the Universe with decelerated expansion ($q>0$) the Hubble's sphere has velocity exceeding the light speed by the quantity $cq$ and thus it overtakes the galaxies situated on its surface. Therefore the galaxies initially situated outside the Hubble's sphere will initially enter inside. Galaxies at distance $R>R_H$ are later $R<R_H$, and their superluminal recession in the course of time becomes subluminal. The light emitted toward the observer by a galaxy outside the Hubble sphere recedes until it enters inside the sphere. Therefore it starts approaching us and becomes available to observations. Therefore, all decelerating Universes lack event horizons unless they terminate at some future time.
 
 
In uniformly expanding Universes $q=0$ the Hubble surface and the galaxies situated on it have equal velocities. Thus number of galaxies in the observable Universe remains constant. Then both particle horizon and event horizon are absent in such Universes.
 
 
In case of the accelerating expansion $q<0$ the Hubble sphere has velocity which is less than the light speed by the quantity $q$ and thus it falls behind the galaxies, and therefore number of them decreases inside the Hubble sphere. All accelerating Universes have the property that galaxies at distance $R<R_H$ are later $R>R_H$, and their subluminal recession in the course of time becomes superluminal. Light emitted outside the Hubble sphere recedes from the observer and can never approach the observer. There are events that can never be observed, and such Universe have event horizons.</p>
 
  </div>
 
</div></div>
 
 
 
<div id="1301_59"></div>
 
<div style="border: 1px solid #AAA; padding:5px;">
 
'''Problem '''
 
<p style= "color: #999;font-size: 11px">problem id: 1301_59</p>
 
Calculate the derivatives $dL_p/dt$ and $dL_e/dt$.
 
<div class="NavFrame collapsed">
 
  <div class="NavHead">solution</div>
 
  <div style="width:100%;" class="NavContent">
 
    <p style="text-align: left;">Using the definitions of the particle horizon \[L_p=a(t)\int\limits_0^t\frac{dt'}{a(t')}\] and the event horizon \[L_e=a(t)\int\limits_t^\infty\frac{dt'}{a(t')},\] one finds
 
\begin{align}
 
\nonumber \frac{dL_p}{dt}=&\frac{d}{dt}\left[a(t)\int\limits_0^t\frac{dt'}{a(t')}\right]=L_p(z)H(z)+1;\\
 
\label{distance_11_3} \frac{dL_e}{dt}=&\frac{d}{dt}\left[a(t)\int\limits_t^\infty\frac{dt'}{a(t')}\right]=L_e(z)H(z)-1.
 
\end{align}</p>
 
  </div>
 
</div></div>
 
 
 
<div id="1301_60"></div>
 
<div style="border: 1px solid #AAA; padding:5px;">
 
'''Problem '''
 
<p style= "color: #999;font-size: 11px">problem id: 1301_60</p>
 
Calculate the derivatives $d^2L_p/dt^2$ and $d^2L_e/dt^2$.
 
<div class="NavFrame collapsed">
 
  <div class="NavHead">solution</div>
 
  <div style="width:100%;" class="NavContent">
 
    <p style="text-align: left;">Differentiating the relations obtained in the previous problem w.r.t. time, one obtains
 
\begin{align}
 
\nonumber \frac{d^2L_p}{dt^2}=&H(1-qHL_p);\\
 
\label{distance_11_5} \frac{dL_e}{dt}=&-H(1+qHL_e).
 
\end{align}</p>
 
  </div>
 
</div></div>
 
 
 
<div id="1301_61"></div>
 
<div style="border: 1px solid #AAA; padding:5px;">
 
'''Problem '''
 
<p style= "color: #999;font-size: 11px">problem id: 1301_61</p>
 
Find the expressions for the current particle horizon in the single-component Universe filled with non-relativistic matter.
 
<div class="NavFrame collapsed">
 
  <div class="NavHead">solution</div>
 
  <div style="width:100%;" class="NavContent">
 
    <p style="text-align: left;">Current value of the proper distance $L_p$ is
 
\[L_p=a_0\int\limits_0^{t_0}\frac{dt'}{a(t')}=a_0\eta|_0^{\eta_0}=a_0\eta_0,\]
 
where $\eta$ is the conformal time.
 
 
Using the expressions for $a_0$ and $\eta_0$ obtained in problems \ref{1301_38} and  \ref{1301_39}, one finds
 
 
\[L_p=a_0\int\limits_0^{r_0}\frac{dr}{\sqrt{1-kr^2}} =a_0\int\limits_0^{t_0}\frac{dt}{a(t)}=\frac1{H_0} \left\{\begin{array}{lcr}
 
2, & k=0, & q_0=1/2;\\
 
\frac{\arcsin{\frac{\sqrt{2q_0-1}}{q_0}}}{\sqrt{2q_0-1}}, & k=1, & q_0>1/2;\\
 
\frac{\mathrm{arcsinh}{\frac{\sqrt{1-2q_0}}{q_0}}}{\sqrt{1-2q_0}}, & k=-1, & q_0<1/2.
 
\end{array}\right.\]</p>
 
  </div>
 
</div></div>
 
 
 
=Power-Law Universes=
 
 
 
 
<div id="1301_62"></div>
 
<div style="border: 1px solid #AAA; padding:5px;">
 
'''Problem '''
 
<p style= "color: #999;font-size: 11px">problem id: 1301_62</p>
 
Assume that the scale factor $a(t)$ varies as $t^n$, where $t$ is the age of Universe and $n=const$. Show that in decelerated expanding Universe $n<1$.
 
<div class="NavFrame collapsed">
 
  <div class="NavHead">solution</div>
 
  <div style="width:100%;" class="NavContent">
 
    <p style="text-align: left;">In these power-law Universes we have
 
\[H\equiv\frac{\dot a}{a}=\frac n t, \quad q\equiv-\frac{\ddot a}{aH^2}=\frac{1-n}t.\]
 
Hence $n<1$ in a decelerating ($q>0$) Universe.</p>
 
  </div>
 
</div></div>
 
 
 
<div id="1301_63"></div>
 
<div style="border: 1px solid #AAA; padding:5px;">
 
'''Problem '''
 
<p style= "color: #999;font-size: 11px">problem id: 1301_63</p>
 
Show that if the scale factor $a(t)$ varies as $t^n$, \[L_p=\frac{R_h}{q}.\]
 
<div class="NavFrame collapsed">
 
  <div class="NavHead">solution</div>
 
  <div style="width:100%;" class="NavContent">
 
    <p style="text-align: left;">In the considered case
 
\[L_p=a(t)\int\limits_0^t\frac{dt'}{a(t')}=\frac{t}{1-n}=\frac t n \frac1q=\frac{R_h}{q}\]</p>
 
  </div>
 
</div></div>
 
 
 
<div id="1301_64"></div>
 
<div style="border: 1px solid #AAA; padding:5px;">
 
'''Problem '''
 
<p style= "color: #999;font-size: 11px">problem id: 1301_64</p>
 
How, in a Universe of age $t$ can causally connected distances of $L\gg ct$ exist?
 
<div class="NavFrame collapsed">
 
  <div class="NavHead">solution</div>
 
  <div style="width:100%;" class="NavContent">
 
    <p style="text-align: left;">We cite below an extremely bright discussion of this question in [E.Harrison, Hubble's spheres and particle horizons, The Astrophisical Journal, 383, 63, 1991]
 
 
" In the study of causal connections, the Hubble sphere bounded by the Hubble surface is as important as the observable universe bounded by the particle horizon.
 
 
Let two comoving bodies be separated by a distance $L$ sufficiently small that each lies in the observable Universe of the other. Each body remains thereafter permanently in the other's observable Universe, and the ratio $L/(ct)$ during expansion depands on the behavior of the Hubble sphere.
 
 
In a  ''decelerating Universe'' the Hubble sphere expands faster than Universe, and a body at distance $L$ either is inside or will soon be inside the Hubble sphere. Hence, any two bodies must eventually recede from each other at subluminal velocity, and the ratio $L/(ct)$ will then decrease in time.
 
 
In an  ''accelerating'' Universe  the Hubble sphere expands slower then universe, and a body at distance $L$ either is outside or will soon be outside the Hubble sphere. Hence, any two bodies must eventually recede from each other at superluminal velocity and the ratio $L/(ct)$ will then increase in time.
 
 
How can causally connected distances of $L\gg ct$ exist? The answeris that the universe passes through a period of accelerated expansion, and causal connections of $L<ct$, established before acceleration, expand superluminally outside the Hubble sphere... A period of accelerated expansion distend all previously established causal connections and increases the distance to the particle horizon."</p>
 
  </div>
 
</div></div>
 
 
 
=The Effects of a Local Expansion of the Universe=
 
 
 
 
<div id="1301_65"></div>
 
<div style="border: 1px solid #AAA; padding:5px;">
 
'''Problem '''
 
<p style= "color: #999;font-size: 11px">problem id: 1301_65</p>
 
Considering the radial motion of a test particle in a spatially-flat expanding Universe find the Newtonian limit the radial force $F$ per unit mass at a distance $R$ from a point mass $m$.
 
<div class="NavFrame collapsed">
 
  <div class="NavHead">solution</div>
 
  <div style="width:100%;" class="NavContent">
 
    <p style="text-align: left;">In order to describe the cosmological expansion one commonly uses two sets of coordinates: the ''physical'' (or Euler) coordinates ($R,\theta,\varphi$) and comoving (or fixed, Lagrangian) coordinates ($r,\theta,\varphi$). (The angular coordinates are the same for both sets as the cosmological expansion is assumed to be radial.) The two sets are related by the formula $R(t)=a(t)r$. Therefore a point which is fixed w.r.t. cosmological expansion, i.e. with constant coordinates ($r,\theta,\varphi$), has additional radial acceleration
 
\begin{equation}\nonumber \left.\frac{d^2R}{dt^2}\right|_{expansion}=R\frac{\ddot a}{a}=-qH^2R.\end{equation}
 
Consequently, in the Newtonian limit the radial force $F$ per unit mass at a distance $R$ from a point mass $m$ is given by
 
\begin{equation}\nonumber F=-\frac{m}{R^2}-q(t)H^2(t)R.\end{equation}
 
The force consists of the usual $1/R^2$ inwards component due to the central (point) mass $m$ and a cosmological component proportional to $R$ that is directed outwards (inwards) when the expansion of the Universe is accelerating (decelerating).</p>
 
  </div>
 
</div></div>
 
 
 
<div id="1301_66"></div>
 
<div style="border: 1px solid #AAA; padding:5px;">
 
'''Problem '''
 
<p style= "color: #999;font-size: 11px">problem id: 1301_66</p>
 
Find the Newtonian limit of the radial force $F$ per unit mass at a distance $R$ from a point mass $m$ in a Universe which contains no matter (or radiation), but only dark energy in the form of a non-zero cosmological constant $\Lambda$.
 
<div class="NavFrame collapsed">
 
  <div class="NavHead">solution</div>
 
  <div style="width:100%;" class="NavContent">
 
    <p style="text-align: left;">In this case, the Hubble parameter and, hence, the deceleration parameter become time-independent and are given by $H=\sqrt{\Lambda/3}$ and $q=-1$. Thus, the force, obtained in the previous problem, also becomes time-independent,
 
\begin{equation}\nonumber F=-\frac{m}{R^2}+\frac13\Lambda R.\end{equation}</p>
 
  </div>
 
</div></div>
 
 
 
<div id="1301_67"></div>
 
<div style="border: 1px solid #AAA; padding:5px;">
 
'''Problem '''
 
<p style= "color: #999;font-size: 11px">problem id: 1301_67</p>
 
Solve the previous problem for the case finite (i.e. non-pointlike) spherically-symmetric massive objects.
 
<div class="NavFrame collapsed">
 
  <div class="NavHead">solution</div>
 
  <div style="width:100%;" class="NavContent">
 
    <p style="text-align: left;">\begin{equation}\nonumber F=-\frac{M(R)}{R^2}+\frac13\Lambda R.\end{equation}
 
where $M(R)$ is the total mass of the object contained within the radius $R$. If the object has the radial density $\rho(R)$ then \[M(R)=\int\limits_0^R4\pi R'^2\rho(R')dR'.\]</p>
 
  </div>
 
</div></div>
 
 
 
<div id="1301_68"></div>
 
<div style="border: 1px solid #AAA; padding:5px;">
 
'''Problem '''
 
<p style= "color: #999;font-size: 11px">problem id: 1301_68</p>
 
Show that a non-zero $\Lambda$ should set a maximum size, dependent on mass, for galaxies and clusters.
 
<div class="NavFrame collapsed">
 
  <div class="NavHead">solution</div>
 
  <div style="width:100%;" class="NavContent">
 
    <p style="text-align: left;">Although the de Sitter background is not an accurate representation of our universe, the SCM is dominated by dark-energy in a form consistent with a simple cosmological constant.  Even in the simple Newtonian case (previous problems), we see immediately that there is an obvious, but profound, difference between the cases $\Lambda=0$ and $\Lambda\ne0$. In the former, the force on a constituent particle of a galaxy or cluster (say) is attractive for all values of $R$ and tends gradually to zero as $R\to\infty$ (for any sensible radial density profile). In the latter case, however, the force on a constituent particle (or equivalently its radial acceleration) vanishes at the finite radius $R_F$ which satisfies $R_F=[3M(R_F)/\Lambda]^{1/3}$, beyond which the net force becomes repulsive. This suggests that a non-zero $\Lambda$ should set a maximum size, dependent on mass, for galaxies and clusters.
 
 
In the Newtonian limit, the speed of a particle in a circular orbit of radius $r$ is given by
 
\[V(R)=\sqrt{\frac{M(R)}{R}-\Lambda R^2},\quad \left(F=-\frac{M(R)}{R^2}+\frac13\Lambda R\right).\]
 
from which it is clear that no circular orbit can exist beyond the radius $R_F$.</p>
 
  </div>
 
</div></div>
 
 
 
<div id="1301_69"></div>
 
<div style="border: 1px solid #AAA; padding:5px;">
 
'''Problem '''
 
<p style= "color: #999;font-size: 11px">problem id: 1301_69</p>
 
The radius $R=R_F$ (see previous problem) does not necessarily correspond to the maximum possible size of the galaxy or cluster:  many of the gravitationally-bound particles inside $R_F$ may be in unstable circular orbits. Therefore the so-called "outer" radius becomes of great importance, which one may interpret as the maximum size of the object, that is corresponding to the largest stable circular orbit $R_S$. Find in the Newtonian limit radius of the largest stable circular orbit $R_S$ in the case of gravity field created by a point mass $m$.
 
<div class="NavFrame collapsed">
 
  <div class="NavHead">solution</div>
 
  <div style="width:100%;" class="NavContent">
 
    <p style="text-align: left;">The radius $R_S$ may be determined as the minimum of the (time-dependent) effective potential for a test particle in orbit about the central mass [R. Nandra, A. N. Lasenby and M. P. Hobson,  The effect of an expanding universe on massive objects, arXiv:1104.4458] . Remaining within the frames of the Newtonian approximation (a weak gravitational field and low velocities), the equation of motion for the test particle is
 
\begin{equation}\nonumber \ddot R\approx-\frac m{R^2}-q(t)H^2(t)R+\frac{L^2}{R^3}.\end{equation}
 
Here $L$ is angular momentum per unit mass. This is simply the Newtonian radial force expression with the inclusion of a centrifugal term.
 
 
The test particle moves in the effective one-dimensional potential $V(R)$
 
\begin{equation}\label{1301_69_e3} V(R)=+\frac{L^2}{2R^2}-\frac m{R}+\frac12q(t)H^2(t)R^2.\end{equation}
 
Extrema of the effective potential in which the test particle moves occur at the $R$-values for which $d^2R/dt^2=0$, namely the solutions of
 
\begin{equation}\nonumber -\frac m{R^2}-q(t)H^2(t)R+\frac{L^2}{R^3}=0.\end{equation}
 
Consider the function
 
\begin{equation}\nonumber y = -mR-q(t)H^2(t)R^4+L^2.\end{equation}
 
The polynomial has single extremum --- a minimum in the point
 
\[R^*=R_S=\left(-\frac m{4q(t)H^2(t)}\right)^{1/3}\]
 
Condition for existence of real roots of the equation (\ref{1301_69_e3}) reads $y(R^*)\le0$. Critical value of the angular momentum for which the minimum of the effective potential disappears can be found from the condition $y(R^*)=0$ and it equals
 
\[L_{crit}=\frac{\sqrt3}{2}m^{1/2}\left(-\frac m{4q(t)H^2(t)}\right)^{1/6}= \left(-\frac{27m^4}{256q(t)H^2(t)}\right)^{1/6}\]</p>
 
  </div>
 
</div></div>
 
 
 
<div id="1301_70"></div>
 
<div style="border: 1px solid #AAA; padding:5px;">
 
'''Problem '''
 
<p style= "color: #999;font-size: 11px">problem id: 1301_70</p>
 
If we consider $R_S(t)$ (see previous problem) as the maximum possible size of a massive object at cosmic time $t$, and assume that the object is spherically-symmetric and has constant density, then it follows that there exists a time-dependent minimum density (due to the maximum size). Determine the corresponding density.
 
<div class="NavFrame collapsed">
 
  <div class="NavHead">solution</div>
 
  <div style="width:100%;" class="NavContent">
 
    <p style="text-align: left;">\[\rho_{min}(t)=\frac{3m}{4\pi R_S^3(t)}=-\frac{3q(t)H^2(t)}{\pi}.\]
 
The latter relation is valid only for $q(t)<0$ (accelerating expansion). For $q(t)>0$, $\rho_{min}(t)=0$.</p>
 
  </div>
 
</div></div>
 
 
 
<div id="1301_71"></div>
 
<div style="border: 1px solid #AAA; padding:5px;">
 
'''Problem '''
 
<p style= "color: #999;font-size: 11px">problem id: 1301_71</p>
 
Show that minimum relative density $\Omega_{min}\equiv\rho_{min}/\rho_{crit}$ is determined only by the deceleration parameter.
 
<div class="NavFrame collapsed">
 
  <div class="NavHead">solution</div>
 
  <div style="width:100%;" class="NavContent">
 
    <p style="text-align: left;">Using result of the previous problem one obtains
 
\[\Omega_{min}\equiv\frac{\rho_{min}}{\rho_{crit}}=-8q(t).\]</p>
 
  </div>
 
</div></div>
 
 
 
<div id="1301_72"></div>
 
<div style="border: 1px solid #AAA; padding:5px;">
 
'''Problem '''
 
<p style= "color: #999;font-size: 11px">problem id: 1301_72</p>
 
An important characteristics -- the minimum fractional density contrast $\delta_{min}(t)\equiv[\rho_{min}(t)-\rho_m(t)]/\rho_m(t)$ is immediately related to the deceleration parameter. Estimate this quantity in the SCM.
 
<div class="NavFrame collapsed">
 
  <div class="NavHead">solution</div>
 
  <div style="width:100%;" class="NavContent">
 
    <p style="text-align: left;">Since $\rho_m(t)=\Omega_m(t)\rho_{crit}(t)$, then
 
\[\delta_{min}(t)=-\left[1+\frac{8q(t)}{\Omega_m(t)}\right].\]
 
For the present time ($\Omega_{m0}\approx0.3$, $q_0\approx-0.55$) one finds that $\delta_{min0}\approx14$.</p>
 
  </div>
 
</div></div>
 
 
-->
 
  
  
Line 1,505: Line 8:
 
'''Problem '''
 
'''Problem '''
 
<p style= "color: #999;font-size: 11px">problem id: 0202_1</p>
 
<p style= "color: #999;font-size: 11px">problem id: 0202_1</p>
Considering the radial motion of a test particle in a spatially-?at expanding Universe show that in the Newtonian limit the radial force $F$ per unit mass at a distance $R$ from a point mass $m$ is given by \[F=-\frac{m}{R^2}-q(t)H^2(t)R.\]
+
Considering the radial motion of a test particle in a spatially-flat expanding Universe show that in the Newtonian limit the radial force $F$ per unit mass at a distance $R$ from a point mass $m$ is given by \[F=-\frac{m}{R^2}-q(t)H^2(t)R.\]
 
<div class="NavFrame collapsed">
 
<div class="NavFrame collapsed">
 
   <div class="NavHead">solution</div>
 
   <div class="NavHead">solution</div>

Revision as of 19:58, 18 June 2015



Problem

problem id: 0202_1

Considering the radial motion of a test particle in a spatially-flat expanding Universe show that in the Newtonian limit the radial force $F$ per unit mass at a distance $R$ from a point mass $m$ is given by \[F=-\frac{m}{R^2}-q(t)H^2(t)R.\]


Problem

problem id: 0202_2

Consider a Universe which contains no matter (or radiation), but only dark energy in the form of a non-zero cosmological constant $\Lambda$. Find the Newtonian limit the radial force $F$ per unit mass at a distance $R$ from a point mass $m$.


Problem

problem id: 0202_3

Derive the expression for the force obtained in the previous problem directly from the Einstein equations.


Problem

problem id: 0202_4

Although the de Sitter background is not an accurate representation of our Universe, the SCM is dominated by dark-energy in a form consistent with a simple cosmological constant. Even in the simple Newtonian case, we see immediately that there is an obvious, but profound, difference between the cases $\Lambda=0$ and $\Lambda\ne0$. In the former, the force on a constituent particle of a galaxy or cluster (say) is attractive for all values of $R$ and tends gradually to zero as $R\to\infty$ (for any sensible radial density profile). In the latter case, however, the force on a constituent particle (or equivalently its radial acceleration) vanishes at the finite radius $R_F$. Find this radius.


Problem

problem id: 0202_5

Show that the structure parameter $R_F$ in general is time-dependent.


Problem

problem id: 0202_6

The radius $R=R_F$ (see previous problem) does not necessarily corresponds to the maximum possible size of the galaxy or cluster: many of the gravitationally-bound particles inside $R_F$ may be in unstable circular orbits. Therefore it is important to know the so-called "outer" radius, which one may interpret as the maximum size of the object, it is the one corresponding to the largest stable circular orbit $R_S$. The radius $R_S$ may be determined as the minimum of the (time-dependent) effective potential for a test particle in orbit about the central mass Show that $R_S=4^{-1/3}R_F$.


Problem

problem id: 0202_7

Find maximum value of the angular momentum $L$ for given values of $M$, $q$ and $H$, at which the stable periodic orbit still exists.


Problem

problem id: 0202_8

If we consider $R_S(t)$ as the maximum possible size of a massive object at cosmic time $t$, and assume that the object is spherically-symmetric and have constant density, then it follows that there exists a time-dependent minimum density (due to maximum size). Find this density.


Problem

problem id: 0202_9

Show that the ratio \(\rho_{min}(t)/\rho_{crit}(t)\) depends only on the deceleration parameter.


Problem

problem id: 0202_10

Estimate for the current moment of time the minimum fractional density $\delta_{min}\equiv[\rho_{min}(t)-\rho_m(t)]/\rho_m(t)$.



New_from_26-12-2014

New from 26-12-2014 New_from_26-12-2014


Exactly Integrable n-dimensional Universes

Exactly Integrable n-dimensional Universes

Problem 1

problem id: gnd_1

Derive Friedmann equations for the spatially n-dimensional Universe.


Problem 2

problem id: gnd_2

Obtain the energy conservation law for the case of n-dimensional Universe.


Problem 3

problem id: gnd_3

Obtain relation between the energy density and scale factor in the case of a two-component n-dimensional Universe dominated by the cosmological constant and a barotropic fluid.


Problem 4

problem id: gnd_4

Obtain equation of motion for the scale factor for the previous problem.


To integrate (\ref{14}), we recall Chebyshev's theorem:

For rational numbers $p,q,r$ ($r\neq0$) and nonzero real numbers $\alpha,\beta$, the integral $\int x^p(\alpha+\beta x^r)^q\,d x$ is elementary if and only if at least one of the quantities \begin{equation}\label{cd} \frac{p+1}r,\quad q,\quad \frac{p+1}r+q, \end{equation} is an integer.

Another way to see the validity of the Chebyshev theorem is to represent the integral of concern by a hypergeometric function such that when a quantity in (\ref{cd}) is an integer the hypergeometric function is reduced into an elementary function. Consequently, when $k=0$ or $\Lambda=0$, and $w$ is rational, the Chebyshev theorem enables us to know that, for exactly what values of $n$ and $w$, the equation (\ref{14}) may be integrated.


Problem 5

problem id: gnd_4_0

Obtain analytic solutions for the equation of motion for the scale factor of the previous problem for spatially flat ($k=0$) Universe.


Problem 6

problem id: gnd_5

Use the analytic solutions obtained in the previous problem to study cosmology with $w>-1$ and $a(0)=0$.


Problem 7

problem id: gnd_6

Use results of the previous problem to calculate the deceleration $q=-a\ddot a/\dot a^2$ parameter.


Problem 8

problem id: gnd_4_1

Obtain the exact solvability conditions for the case $\Lambda=0$ in equation (\ref{14}) (see problem #gnd_4 ).


Problem 9

problem id: gnd_4_2

Obtain the explicit solutions for the case $n=3$ and $w=-5/9$ in the previous problem.


Problem 10

problem id: gnd_10

Rewrite equation of motion for the scale factor (\ref{14}) in terms of the conformal time.


Problem 11

problem id: gnd_11

Find the rational values of the equation of state parameter $w$ which provide exact integrability of the equation (\ref{48}) with $k=0$.


Problem 12

problem id: gnd_12

Obtain explicit solutions for the case $w=\frac1n-1$ in the previous problem.


Problem 13

problem id: gnd_13

Obtain exact solutions for the equation (\ref{48}) with $\Lambda=0$.


Problem 14

problem id: gnd_14

Obtain inflationary solutions using the results of the previous problem.


UNSORTED NEW Problems

The history of what happens in any chosen sample region is the same as the history of what happens everywhere. Therefore it seems very tempting to limit ourselves with the formulation of Cosmology for the single sample region. But any region is influenced by other regions near and far. If we are to pay undivided attention to a single region, ignoring all other regions, we must in some way allow for their influence. E. Harrison in his book Cosmology, Cambridge University Press, 1981 suggests a simple model to realize this idea. The model has acquired the name of "Cosmic box" and it consists in the following.

Imaginary partitions, comoving and perfectly reflecting, are used to divide the Universe into numerous separate cells. Each cell encloses a representative sample and is sufficiently large to contain galaxies and clusters of galaxies. Each cell is larger than the largest scale of irregularity in the Universe, and the contents of all cells are in identical states. A partitioned Universe behaves exactly as a Universe without partitions. We assume that the partitions have no mass and hence their insertion cannot alter the dynamical behavior of the Universe. The contents of all cells are in similar states, and in the same state as when there were no partitions. Light rays that normally come from very distant galaxies come instead from local galaxies of long ago and travel similar distances by multiple re?ections. What normally passes out of a region is reflected back and copies what normally enters a region.

Let us assume further that the comoving walls of the cosmic box move apart at a velocity given by the Hubble law. If the box is a cube with sides of length $L$, then opposite walls move apart at relative velocity $HL$.Let us assume that the size of the box $L$ is small compared to the Hubble radius $L_{H} $ , the walls have a recession velocity that is small compared to the velocity of light. Inside a relatively small cosmic box we use ordinary everyday physics and are thus able to determine easily the consequences of expansion. We can even use Newtonian mechanics to determine the expansion if we embed a spherical cosmic box in Euclidean space.


Problem 1

problem id:

As we have shown before (see Chapter 3): $p(t)\propto a(t)^{-1} $ , so all freely moving particles, including galaxies (when not bound in clusters), slowly lose their peculiar motion and ultimately become stationary in expanding space. Try to understand what happens by considering a moving particle inside an expanding cosmic box


Problem 2

problem id:

Show that at redshift $z=1$ , when the Universe is half its present size, the kinetic energy of a freely moving nonrelativistic particle is four times its present value, and the energy of a relativistic particle is twice its present value.


Problem 3

problem id:

Let the cosmic box is filled with non-relativistic gas. Find out how the gas temperature varies in the expanding cosmic box.


Problem 4

problem id:

Show that entropy of the cosmic box is conserved during its expansion.


Problem 5

problem id:

Consider a (cosmic) box of volume V, having perfectly reflecting walls and containing radiation of mass density $\rho $. The mass of the radiation in the box is $M=\rho V$ . We now weigh the box and find that its mass, because of the enclosed radiation, has increased not by M but by an amount 2M. Why?


Problem 6

problem id:

Show that the jerk parameter is \[j(t)=q+2q^{2} -\frac{\dot{q}}{H} \]


Problem 7

problem id:

We consider FLRW spatially flat Universe with the general Friedmann equations \[\begin{array}{l} {H^{2} =\frac{1}{3} \rho +f(t),} \\ {\frac{\ddot{a}}{a} =-\frac{1}{6} \left(\rho +3p\right)+g(t)} \end{array}\] Obtain the general conservation equation.


Problem 8

problem id:

Show that for extra driving terms in the form of the cosmological constant the general conservation equation (see previous problem) transforms in the standard conservation equation.


Problem 9

problem id:

Show that case $f(t)=g(t)=\Lambda /3$ corresponds to $\Lambda (t)CDM$ model.